<<

Online Prelims Thematic TEST - 10 ( InsightsIAS Mock Test Series for UPSC Preliminary Exam 2020 )

1 ‘RNA interference (RNAi)’ technology is being used to suppress desired genes. Which of the following is/are examples of such an application? 1. Decaffeinated Coffee 2. Nicotine free tobacco 3. Hypoallergenic crops

Select the correct answer using the code given below. A. 1 only B. 2 and 3 C. 1 and 3 D. 1, 2 and 3

Your Answer : Correct Answer : D

Answer Justification :

Justification: RNA interference (RNAi) is a biological process in which RNA molecules inhibit gene expression or translation.

Since the of RNAi and its regulatory potentials, it has become evident that RNAi has immense potential in suppression of desired genes (gene silencing).

Numerous studies have demonstrated that RNAi can provide a more specific approach to inhibit tumor growth by targeting cancer-related genes (i.e., oncogene).

RNAi has resulted in the invention of novel crops such as nicotine-free tobacco, decaffeinated coffee, nutrient fortified vegetation, and hypoallergenic crops (allergen free crops). The genetically-engineered Arctic received FDA approval in 2015.

While it was known that plants expressing virus-specific proteins showed enhanced tolerance or resistance to viral infection, it was not expected that plants carrying only short, non-coding regions of viralIMAGERUNNERS RNA sequences would 8860450330show similar levels of protection. Researchers believed that viral RNA produced by transgenes could also inhibit viral replication.

The reverse experiment, in which short sequences of plant genes were introduced into viruses, showed that the targeted gene was suppressed in an infected plant. This phenomenon was labeled “virus-induced gene silencing” (VIGS), and the set of such phenomena were collectively called post transcriptional gene silencing.

See https://en.wikipedia.org/wiki/RNA_interference

Q Source: AR: UPSC CSP 2019

2 Consider the following statements: 1. The United Nations Convention against Corruption is the only legally binding international anti- corruption multilateral treaty. 2. The Indian Government has ratified the United Nations Convention against Corruption (UNCAC). prelims.insightsonindia.com 1 © Insights Active Learning | All rights reserved - 133617. You may not reproduce, distribute or exploit the contents in any form without written permission by copyright owner. Copyright infringers may face civil and criminal liability Online Prelims Thematic TEST - 10 ( InsightsIAS Mock Test Series for UPSC Preliminary Exam 2020 )

3. The Protocol against the Illicit Manufacturing of and Trafficking in Firearms, their Parts and Components and Ammunition is a protocol under UNCAC. 4. No south Asian country has so far ratified United Nations Convention against Transnational Organised Crime (UNTOC).

Which of the statements given above are correct? A. 1 and 4 only B. 1 and 2 only C. 2, 3 and 4 only D. 1, 2, 3 and 4

Your Answer : Correct Answer : B

Answer Justification :

Justification: The United Nations Convention against Corruption is the only legally binding international anti-corruption multilateral treaty.

In May 2011, the Indian Government ratified two UN Conventions - the United Nations Convention against Corruption (UNCAC) and the United Nations Convention against Transnational Organised Crime (UNTOC) and its three protocols.

Having ratified both Conventions, India became the fourth South Asian country after Afghanistan, Pakistan and Sri Lanka to ratify the UNTOC while joining Afghanistan, Bangladesh, Maldives, Nepal, Pakistan and Sri Lanka in ratifying the UNCAC. The United Nations Office on Drugs and Crime (UNODC) is mandated by its Member States to assist in the implementation of both Conventions, which along with the UN Drug Conventions of 1961, 1971 and 1988 underpin all the operational work of UNODC.

The UNTOC is further supplemented by three Protocols, which target specific forms of organized crime: IMAGERUNNERS 8860450330 1) The Protocol to Prevent, Suppress and Punish Trafficking in Persons, Especially Women and Children, provides an agreed upon definition of trafficking in persons. It aims at comprehensively addressing trafficking in persons through the so-called three P's - Prosecution of perpetrators, Protection of victims and Prevention of trafficking.

2) The Protocol against the Smuggling of Migrants by Land, Sea and Air, also provides a definition of smuggling of migrants. The Protocol aims at preventing and controlling smuggling of migrants, promoting cooperation among States Parties, while protecting the rights of smuggled migrants.

3) The Protocol against the Illicit Manufacturing of and Trafficking in Firearms, their Parts and Components and Ammunition promotes, facilitates and strengthens cooperation among States Parties in order to prevent and control the illicit manufacturing of and trafficking in firearms, their parts and components and ammunition by mainly committing Member States to introduce solid registration and storage systems for all legally produced arms.

prelims.insightsonindia.com 2 © Insights Active Learning | All rights reserved - 133617. You may not reproduce, distribute or exploit the contents in any form without written permission by copyright owner. Copyright infringers may face civil and criminal liability Online Prelims Thematic TEST - 10 ( InsightsIAS Mock Test Series for UPSC Preliminary Exam 2020 )

The United Nations Office on Drugs and Crime (UNODC) is mandated by its Member States to assist in the implementation of both Conventions, which along with the UN Drug Conventions of 1961, 1971 and 1988 underpin all the operational work of UNODC.

See https://www.unodc.org/southasia/en/frontpage/2011/may/indian-govt-ratifies-two-un-convention s.html

Q Source: AR: UPSC CSP 2019

3 Consider the following statements. 1. The Service Area Approach was implemented under the purview of Lead Bank Scheme. 2. SAA was extended to all Indian scheduled commercial banks including Regional Rural Banks (RRBs).

Which of the above is/are correct? A. 1 only B. 2 only C. Both 1 and 2 D. None

Your Answer : Correct Answer : C

Answer Justification :

Justification: Service area approach (SAA) is a developed version of the ‘area approach’ structure of the Lead Bank Scheme.

Under SAA plan each commercial bank / RRB branch in a rural and semi-urban area is designated to serve 15 to 25 villages for the planned and orderly development of the areas.

The designated branch of a bank has to meet the banking needs of its service area vis-à-vis forge effective IMAGERUNNERS linkages between bank 8860450330 credit, production, productivity, and an increase in income levels of the villages.

SAA was introduced in April 1989 in order to bring about an orderly and planned development of rural and semi- urban areas of the country.

See https://rbi.org.in/scripts/NotificationUser.aspx?Id=2044&Mode=0

Q Source: AR: UPSC 2019

4 Which of the following acts exempts several constitutional/public posts from disqualification on the grounds of ‘Office of Profit’?

A. Representation of People Act, 1950 B. Representation of People Act, 1951 C. The Parliament (Prevention of Disqualification) Act, 1959 prelims.insightsonindia.com 3 © Insights Active Learning | All rights reserved - 133617. You may not reproduce, distribute or exploit the contents in any form without written permission by copyright owner. Copyright infringers may face civil and criminal liability Online Prelims Thematic TEST - 10 ( InsightsIAS Mock Test Series for UPSC Preliminary Exam 2020 )

D. None of the above

Your Answer : Correct Answer : C

Answer Justification :

Justification: According to Articles 102(1)a) and 191(1)a) of Constitution, legislators (MP or MLA) can be barred from holding office of profit under Central Government or state government as it can put them in position to gain financial benefit.

However, none of the following offices, in so far as it is an office of profit under the Government of India or the Government of any State, shall disqualify the holder thereof for being chosen as, or for being, a member of Parliament, namely:--- (a) any office held by a Minister, Minister of State or Deputy Minister for the Union or for any State, whether ex officio or by name:

(b) the office of Chief Whip, Deputy Chief Whip or Whip in Parliament or of a Parliamentary Secretary;

(c) the office of a member of any force raised or maintained under the National Cadet Corps Act, 1948 (31 of 1948.), the Territorial Army Act, 1948 (56 of 1948.), or the Reserve and Auxiliary Air Forces Act, 1952 (62 of 1952);

(d) the office of a member of a Home Guard constituted under any law for the time being in force in any State; etc.

See more here http://theindianlawyer.in/statutesnbareacts/acts/p2.html

Q Source: Based on UPSC papers

5 Consider the following statements: 1. The largest chunk of India’s external debt comes from bilateral and multilateral debt owed to nations IMAGERUNNERS and international financial 8860450330 institutions. 2. Most of India’s external debt is denominated in Special Drawing Rights (SDRs).

Which of the statements given above is / are correct? A. 1 only B. 2 only C. Both 1 and 2 D. Neither 1 nor 2

Your Answer : Correct Answer : D

Answer Justification :

Justification: S1: The largest portion are ECBs (commercial borrowings). Bilateral debt is the money India owes to foreign governments and Multilateral debt is the money India owes to prelims.insightsonindia.com 4 © Insights Active Learning | All rights reserved - 133617. You may not reproduce, distribute or exploit the contents in any form without written permission by copyright owner. Copyright infringers may face civil and criminal liability Online Prelims Thematic TEST - 10 ( InsightsIAS Mock Test Series for UPSC Preliminary Exam 2020 )

international financial institutions such as the ADB or IMF. Clearly they are not the largest chunks.

S2: India’s external debt is held in multiple currencies, the largest of which is the United States dollar. As on 31 December 2017, 48.2% of the country’s debt was held in U.S. dollars. The rest of the debt is held in Indian rupees (37.3%), special drawing rights (5.7%), Japanese yen (4.6%), Euros (3.2%) and other currencies (1%), see https://www.rbi.org.in/scripts/BS_PressReleaseDisplay.aspx?prid=44350 and https://en.wikipedi a.org/wiki/External_debt_of_India

Q Source: AR: UPSC 2019

6 Which of the following commodities were exported by the English East India Company from Bengal in the middle of the 18th century? 1. Oil-seeds 2. Tea 3. Saltpetre 4. Silk

Which of the statements given above are correct? A. 1, 2 and 4 only B. 2 and 4 only C. 1, 3 and 4 only D. 2, 3 and 4 only

Your Answer : Correct Answer : D

Answer Justification :

Justification: The East India Company was originally formed in Britain for pursuing trade with the East Indies in Southeast Asia.

In fact,IMAGERUNNERS it ended up trading mainly 8860450330 with the Indian subcontinent and China, where the main items of trade were cotton, silk, tea, opium, and saltpetre (potassium nitrate).

See https://en.wikipedia.org/wiki/East_India_Company

Q Source: Based on UPSC papers

7 Consider the following about Jahangir’s promotion of painting. 1. He banished European style of painting from his . 2. He promoted the flattened multilayered style used in traditional miniature paintings as against the single-point perspective found in European paintings.

Which of the statements given above are correct? A. 1 only B. 2 only prelims.insightsonindia.com 5 © Insights Active Learning | All rights reserved - 133617. You may not reproduce, distribute or exploit the contents in any form without written permission by copyright owner. Copyright infringers may face civil and criminal liability Online Prelims Thematic TEST - 10 ( InsightsIAS Mock Test Series for UPSC Preliminary Exam 2020 )

C. Both 1 and 2 D. None

Your Answer : Correct Answer : D

Answer Justification :

Justification: Jahangir was deeply influenced by European painting. During his reign he came into direct contact with the English Crown and was sent gifts of oil paintings, which included portraits of the King and Queen.

He encouraged his royal atelier to take up the single point perspective favoured by European artists, unlike the flattened multi-layered style used in traditional miniatures. He particularly encouraged paintings depicting events of his own life, individual portraits, and studies of birds, flowers and animals.

See https://en.wikipedia.org/wiki/Mughal_painting#Jahangir_(1605%E2%80%9325)

Q Source: Based on UPSC past year papers

8 Consider the following about India’s agricultural export data of the last five years. 1. India is the largest agricultural exporter of the World in terms of total economic value. 2. Among the agricultural commodities imported by India, vegetable oils account for the highest import in terms of total value. 3. Among the agricultural commodities exported by India, Guar gum account for the highest exports in terms of total value.

Which of the above is/are correct? A. 1 and 3 only B. 2 only C.IMAGERUNNERS1 and 2 only 8860450330 D. 2 and 3 only

Your Answer : Correct Answer : B

Answer Justification :

Justification: India has long been recognized as an agriculture powerhouse, but has performed much below its potential when it comes to agricultural exports. In spite of being the number one producer of dairy, mango, banana and second largest producer of cereals, fruits and vegetables, India ranks 10th among the countries with highest agricultural exports. Small countries like Belgium, Italy, Netherlands, etc. export much higher value of agricultural goods than us.

In FY19, export of agricultural and processed food products totalled US$ 38.49 billion. During the period, top commodities to be exported were basmati rice (US$ 4.71 billion), buffalo meat (US$ 3.58 billion) and non-basmati rice (US$ 3.00 billion). prelims.insightsonindia.com 6 © Insights Active Learning | All rights reserved - 133617. You may not reproduce, distribute or exploit the contents in any form without written permission by copyright owner. Copyright infringers may face civil and criminal liability Online Prelims Thematic TEST - 10 ( InsightsIAS Mock Test Series for UPSC Preliminary Exam 2020 )

As for imports, refer to the table below: Vegetable oils formed the largest chunk of agricultural imports:

See https://commerce.gov.in/writereaddata/uploadedfile/MOC_636626711232248483_Annual%20Re port%20%202017-18%20English.pdf , page 39

Q Source: AR: UPSC 2019

9 Kanganahalli is considered an important Buddhist site because 1. an ancientIMAGERUNNERS Buddhist Mahastupa 8860450330 site was excavated here 2. several sculptured panels depicting various Jataka stories could be found in excavations here

Select the correct answer using the code given below. A. 1 only B. 2 only C. Both 1 and 2 D. None

Your Answer : Correct Answer : C

Answer Justification :

Justification: Kanganahalli is an important Buddhist site, the place where an ancient Buddhist Mahastupa site found. It is on the left bank of the Bhima river in Chitapur taluk, Gulbarga District in prelims.insightsonindia.com 7 © Insights Active Learning | All rights reserved - 133617. You may not reproduce, distribute or exploit the contents in any form without written permission by copyright owner. Copyright infringers may face civil and criminal liability Online Prelims Thematic TEST - 10 ( InsightsIAS Mock Test Series for UPSC Preliminary Exam 2020 )

Karnataka.

Excavation during 1994 to 1998

During the excavations (1994 to 1998) at Kanaganahalli, found remains of a massive , many brick structures in the form of a Chaitya-griha and votive were brought to light. During the excavations many architectural members of the stupa found like fragments of sculptured veneering slabs, members of railings, pillars, capitals, Buddha padas, sculptures of yaksha and four images of Buddha many more. The sculptured panels depicts various Jataka stories and life of Lord Buddha and portrait of many Shatavahana king.

Inscriptions

In addition to one long inscription, 145 short inscriptions were discovered from the excavations site, dating between 1st century BCE to 1st century CE. The very important discovery was the sculpture of Maurya emperor Ashoka with the label "Rayo Asoka".

Kanaganahalli in Karnataka is the site with an inscription in Brahmi script reading "Ranyo Ashoka" (King Ashoka) and a sculpture of King Ashoka.

This was the first inscribed portrait of Ashoka (surrounded by female attendants and queens) that was unearthed from the ruined Buddhist stupa.

See See https://en.wikipedia.org/wiki/Kanaganahalli#Excavation_during_2000_to_2002

Q Source: AR: UPSC CSP 2019

10 Which of these are thermal processes that use high temperatures to break down waste to create energy? 1. Pyrolysis 2. Plasma gasification 3. Thermal IMAGERUNNERS depolymerization 8860450330 4. Anaerobic digestion

Which of the above is/are correct? A. 1, 2 and 3 only B. 2 and 4 only C. 1 and 2 only D. 1, 2, 3 and 4

Your Answer : Correct Answer : A

Answer Justification :

Justification: There are a number of other new and emerging technologies that are able to produce energy from waste and other fuels without direct combustion. Many of these technologies have the potential to produce more electric power from the same amount of fuel than would be possible by prelims.insightsonindia.com 8 © Insights Active Learning | All rights reserved - 133617. You may not reproduce, distribute or exploit the contents in any form without written permission by copyright owner. Copyright infringers may face civil and criminal liability Online Prelims Thematic TEST - 10 ( InsightsIAS Mock Test Series for UPSC Preliminary Exam 2020 )

direct combustion. This is mainly due to the separation of corrosive components (ash) from the converted fuel, thereby allowing higher combustion temperatures in e.g. boilers, gas turbines, internal combustion engines, fuel cells. Some are able to efficiently convert the energy into liquid or gaseous fuels:

Thermal technologies:

Gasification: produces combustible gas, hydrogen, synthetic fuels Thermal depolymerization: produces synthetic crude oil, which can be further refined Pyrolysis: produces combustible tar/biooil and chars Plasma arc gasification or plasma gasification process (PGP): produces rich syngas including hydrogen and carbon monoxide usable for fuel cells or generating electricity to drive the plasma arch, usable vitrified silicate and metal ingots, salt and sulphur

Like incineration, pyrolysis, gasification and plasma technologies are thermal processes that use high temperatures to break down waste. The main difference is that they use less oxygen than traditional mass-burn incineration.

Non-thermal technologies:

Anaerobic digestion: Biogas rich in methane Fermentation production: examples are ethanol, lactic acid, hydrogen Mechanical biological treatment (MBT) MBT + Anaerobic digestion MBT to Refuse derived fuel

See https://friendsoftheearth.uk/sites/default/files/downloads/gasification_pyrolysis.pdf

Q Source: AR: UPSC CSP 2019

11 Consider the following statements: 1. The government has notified fixed term employment for all sectors through an amendment to the Industrial IMAGERUNNERS Employment (Standing 8860450330 Orders) Central Rules, 1946. 2. As per the Industrial Employment (Standing Orders) Central (Amendment) Rules, 2018, a “fixed term employment workman is a workman who has been engaged on the basis of a written contract of employment for a fixed period”.

Which of the statements given above is / are correct? A. 1 only B. 2 only C. Both 1 and 2 D. Neither 1 nor 2

Your Answer : Correct Answer : C

Answer Justification :

prelims.insightsonindia.com 9 © Insights Active Learning | All rights reserved - 133617. You may not reproduce, distribute or exploit the contents in any form without written permission by copyright owner. Copyright infringers may face civil and criminal liability Online Prelims Thematic TEST - 10 ( InsightsIAS Mock Test Series for UPSC Preliminary Exam 2020 )

The government has notified fixed term employment for all sectors through an amendment to the Industrial Employment (Standing Orders) Central Rules, 1946.

1. As per the Industrial Employment (Standing Orders) Central (Amendment) Rules, 2018, a “fixed term employment workman is a workman who has been engaged on the basis of a written contract of employment for a fixed period”. 2. No workman employed on fixed term employment basis as a result of non-renewal of contract or employment or on its expiry shall be entitled to any notice or pay in lieu thereof, if his services are terminated. No notice of termination of employment shall be necessary in the case of temporary workman whether monthly rated, weekly rated or piece rated and probationers or badli workmen 3. Therefore, fixed-term employment for all sectors will make it easier for companies to hire-and- fire workers along with reducing the role of middlemen.

No notice of termination of employment shall be necessary in the case of temporary workman whether monthly rated, weekly rated or piece rated and probationers or badli workmen.

Q Source: AR: UPSC CSP 2019

12 Aralam, Mudumalai, Mukurthi, Nagarhole, Bandipur and Silent Valley national parks are under which of these Biosphere Reserves?

A. Agasthyamala B. Nilgiri C. Wayanad D. Seshachalam

Your Answer : Correct Answer : B

Answer Justification :

Justification: IMAGERUNNERS The Nilgiri Biosphere 8860450330 Reserve is an International Biosphere Reserve in the Western Ghats and Nilgiri Hills ranges of South India. The Nilgiri Sub-Cluster is a part of the Western Ghats, which was declared a World Heritage Site by UNESCO in 2012. It includes the Aralam, Mudumalai, Mukurthi, Nagarhole, Bandipur and Silent Valley national parks, as well as the Wayanad and Sathyamangalam wildlife sanctuaries.

Nilgiris Biosphere Reserve is India's first and foremost biosphere reserves with a heritage, rich in flora and fauna. Tribal groups like the Badagas, Todas, Kotas, Irullas, Kurumbas, Paniyas, Adiyans, Edanadan Chettis, Allar, Malayan, etc., are native to the reserve

See http://www.kerenvis.nic.in/Database/BiosphereReservesinKerala_1293.aspx

Q Source: Based on several UPSC papers

13 The National Council of Education (NCE) was an organisation founded by Indian nationalists in Bengal in the event of which of the following movements? prelims.insightsonindia.com 10 © Insights Active Learning | All rights reserved - 133617. You may not reproduce, distribute or exploit the contents in any form without written permission by copyright owner. Copyright infringers may face civil and criminal liability Online Prelims Thematic TEST - 10 ( InsightsIAS Mock Test Series for UPSC Preliminary Exam 2020 )

A. Non-cooperation movement B. Swadeshi movement C. Civil Disobedience movement D. Quit India movement

Your Answer : Correct Answer : B

Answer Justification :

Justification: The National Council of Education (or NCE) was an organisation founded by Indian nationalists in Bengal in 1906 to promote science and technology as part of a swadeshi industrialisation movement.

It established the Bengal National College and Bengal Institute which would later merge to form Jadavpur University.

See more here https://en.wikipedia.org/wiki/National_Council_of_Education

Q Source: Based on UPSC CSP 2019

14 Wearable technology is being incorporated these days into 1. Navigation systems 2. Advanced textiles 3. Healthcare systems

Select the correct answer using the code given below. A. 1 only B. 2 and 3 only C. 3 only D. 1, 2 and 3 IMAGERUNNERS 8860450330 Your Answer : Correct Answer : D

Answer Justification :

Justification: Wearable technology, wearables, fashion technology, tech togs, or fashion electronics are smart electronic devices (electronic device with micro-controllers) that are worn close to and/or on the surface of the skin, where they detect, analyze, and transmit information concerning e.g. body signals such as vital signs, and/or ambient data and which allow in some cases immediate biofeedback to the wearer.

Wearable technology appears prominently in consumer electronics with the popularization of the smartwatch and activity tracker. Apart from commercial uses, wearable technology is being incorporated into navigation systems, advanced textiles, and healthcare. Hearing aid wearable devices are more common and older than the smartwatches popular today.

prelims.insightsonindia.com 11 © Insights Active Learning | All rights reserved - 133617. You may not reproduce, distribute or exploit the contents in any form without written permission by copyright owner. Copyright infringers may face civil and criminal liability Online Prelims Thematic TEST - 10 ( InsightsIAS Mock Test Series for UPSC Preliminary Exam 2020 )

Wearable technology has a variety of applications which grows as the field itself expands. It appears prominently in consumer electronics with the popularization of the smartwatch and activity tracker. Apart from commercial uses, wearable technology is being incorporated into navigation systems, advanced textiles, and healthcare.

See https://en.wikipedia.org/wiki/Wearable_technology

Q Source: AR: UPSC CSP 2019

15 Petroleum and Natural Gas Regulatory Board (PNGRB) performs which of the following functions? 1. Enforce service obligations for retail outlets 2. Monitor transportation rates of petroleum in India 3. Take corrective action to prevent restrictive trade practice by the involved trading entities

Which of the statements given above are correct? A. 1 and 2 only B. 2 and 3 only C. 1 and 3 only D. 1, 2 and 3

Your Answer : Correct Answer : D

Answer Justification :

Justification: PGNRB was constituted in 2006.

The Board shall-

(A) Protect the interest of consumers by fostering fair trade and competition amongst the entities;

(B) Register entities to- IMAGERUNNERS 8860450330 1. market notified petroleum and petroleum products and, subject to the contractual obligations of the Central Government, natural gas; 2. establish and operate liquefied natural gas terminals; 3. establish storage facilities for petroleum, petroleum products or natural gas exceeding such capacity as may be specified by regulations;

(C) Authorise entities to-

1. lay, build, operate or expand a common carrier or contract carrier; 2. lay, build, operate or expand city or local natural gas distribution network;

(D) Declare pipelines as common carrier or contract carrier;

(E) Regulate, by regulations,-

1. access to common carrier or contract carrier so as to ensure fair trade and competition prelims.insightsonindia.com 12 © Insights Active Learning | All rights reserved - 133617. You may not reproduce, distribute or exploit the contents in any form without written permission by copyright owner. Copyright infringers may face civil and criminal liability Online Prelims Thematic TEST - 10 ( InsightsIAS Mock Test Series for UPSC Preliminary Exam 2020 )

amongst entities and for that purpose specify pipeline access code; 2. transportation rates for common carrier or contract carrier; 3. access to city or local natural gas distribution network so as to ensure fair trade and competition amongst entities as per pipeline access code;

(F) In respect of notified petroleum, petroleum products and natural gas-

1. ensure adequate availability; 2. ensure display of information about the maximum retail prices fixed by the entity for consumers at retail outlets; 3. monitor prices and take corrective measures to prevent restrictive trade practice by the entities; 4. secure equitable distribution for petroleum and petroleum products; 5. provide, by regulations, and enforce, retail service obligations for retail outlets and marketing service obligations for entities; 6. monitor transportation rates and take corrective action to prevent restrictive trade practice by the entities;

(G) Levy fees and other charges as determined by regulations;

(H) Maintain a data bank of information on activities relating to petroleum, petroleum products and natural gas;

(I) Lay down, by regulations, the technical standards and specifications including safety standards in activities relating to petroleum, petroleum products and natural gas, including the construction and operation of pipeline and infrastructure projects related to downstream petroleum and natural gas sector;

Q Source: AR: UPSC 2019

16 Consider the following statements. 1. The Commission on Narcotic Drugs (CND) and the Commission on Crime Prevention and Criminal Justice IMAGERUNNERS (CCPCJ) were created by8860450330 resolutions of the UN Economic and Social Council. 2. The CND and CCPCJ are the Governing Bodies of the United Nations Office on Drugs and Crime (UNODC).

Which of the above is/are correct? A. 1 only B. 2 only C. Both 1 and 2 D. None

Your Answer : Correct Answer : C

Answer Justification :

Learning: The Commission on Narcotic Drugs (CND) and the Commission on Crime Prevention and prelims.insightsonindia.com 13 © Insights Active Learning | All rights reserved - 133617. You may not reproduce, distribute or exploit the contents in any form without written permission by copyright owner. Copyright infringers may face civil and criminal liability Online Prelims Thematic TEST - 10 ( InsightsIAS Mock Test Series for UPSC Preliminary Exam 2020 )

Criminal Justice (CCPCJ) are policymaking bodies within the United Nations system and guide international action against drugs and crime.

The CND and CCPCJ are functional commissions of the United Nations Economic and Social Council (ECOSOC) [also created by its resolutions] and Governing Bodies of the United Nations Office on Drugs and Crime (UNODC).

Their resolutions and decisions provide guidance in their respective areas to Member States and the UNODC. The thematic areas covered by the CND and CCPCJ are also dealt with by the General Assembly, in particular its Third Committee, which deals with Social, Humanitarian and Cultural Affairs, and its Fifth Committee, which deals with budgetary matters relating to the governing bodies functions.

See https://www.unodc.org/unodc/en/commissions/CCPCJ/index.html

Q Source: AR: UPSC CSP 2019

17 Which of these actions can lead to the growth of money multiplier in an economy?

A. Promoting the use of credit cards B. Public opting for more loans C. An increase in the cash reserve ratio D. Both (a) and (b)

Your Answer : Correct Answer : D

Answer Justification :

Justification: We have covered this concept in previous tests, so only a brief given here.

Option C: Any increase in a reserve ratio prevents the banks from lending more money, and reduces the IMAGERUNNERSmoney multiplier. 8860450330

Option A and B: Promoting the use of credit cards, taking more loans etc. can lead to lending and re-lending of the same money supply several times increasing the money multiplier.

Q Source: AR: UPSC CSP 2019

18 Which of the above is/are a part of the belief systems of Mahayana Buddhism? 1. Buddha-fields do not exist. 2. reside in different worlds. 3. Inferior and superior exist as per disciple’s capabilities.

Select the correct answer using the codes below. A. 1 only B. 1 and 2 only C. 2 and 3 only prelims.insightsonindia.com 14 © Insights Active Learning | All rights reserved - 133617. You may not reproduce, distribute or exploit the contents in any form without written permission by copyright owner. Copyright infringers may face civil and criminal liability Online Prelims Thematic TEST - 10 ( InsightsIAS Mock Test Series for UPSC Preliminary Exam 2020 )

D. 1, 2 and 3

Your Answer : Correct Answer : C

Answer Justification :

Justification: S3: [A]ccording to disciples' grades, the Dharma is [classified as] inferior and superior. For example, the inferior was taught to the merchants Trapuṣa and Ballika because they were ordinary men; the middle was taught to the group of five because they were at the stage of saints; the eightfold Prajñāpāramitās were taught to bodhisattvas, and [the Prajñāpāramitās] are superior in eliminating conceptually imagined forms.

S1 and S2: Buddhas and bodhisattvas are central elements of Mahāyāna. Mahāyāna’s vastly expanded cosmology, with various Buddhas and bodhisattvas residing in different worlds and buddha-fields.

This is shown through the depiction of buddhas and bodhisattvas through image worship and rituals in monasteries and viharas. Popular bodhisattvas include Avalokiteshvara, Manjushri and Maitreya. Bodhisattvas could reach the personal nirvana of the arhats, but they believe it is more important to remain in saṃsāra and help others

See https://en.wikipedia.org/wiki/Mahayana#Doctrine

Q Source: AR: UPSC CSP 2019

19 With reference to Gupta period, consider the following statements. 1. The texts Brihaspati Smriti and Narada Smriti lay down rules for the payment of wages to labour in cash or kind. 2. Forced labour was not to be employed, either by the State or privately, as prescribed by the Narada Smriti.

Which IMAGERUNNERS of the above is/are correct? 8860450330 A. 1 only B. 2 only C. Both 1 and 2 D. None

Your Answer : Correct Answer : A

Answer Justification :

Justification: We are attaching a screenshot from the History book by Upinder Singh, a favourite of UPSC.

prelims.insightsonindia.com 15 © Insights Active Learning | All rights reserved - 133617. You may not reproduce, distribute or exploit the contents in any form without written permission by copyright owner. Copyright infringers may face civil and criminal liability Online Prelims Thematic TEST - 10 ( InsightsIAS Mock Test Series for UPSC Preliminary Exam 2020 )

See https://tinyurl.com/y6naszkx

Q Source: AR: UPSC CSP 2019

20 Consider the following statements about climate engineering techniques. 1. Stratospheric sulphate aerosols can create a global dimming effect. 2. Cirrus cloud thinning is a proposed form of climate engineering because it increases their heat trapping capacity.

Which of the above is/are correct? A. 1 only B. 2 only C. Both 1 and 2 D.IMAGERUNNERSNone 8860450330

Your Answer : Correct Answer : A

Answer Justification :

Justification: The ability of stratospheric sulfate aerosols to create a global dimming effect has made them a possible candidate for use in solar radiation management climate engineering projects to limit the effect and impact of climate change due to rising levels of greenhouse gases. Delivery of precursor sulfide gases such as sulfuric acid, hydrogen sulfide (H2S) or sulfur dioxide (SO2) by artillery, aircraft and balloons has been proposed.

Cirrus cloud thinning is a proposed form of climate engineering. Cirrus clouds are high cold ice that, like other clouds, both reflect sunlight and absorb warming infrared radiation. However, they differ from other types of clouds in that, on average, infrared absorption outweighs sunlight prelims.insightsonindia.com 16 © Insights Active Learning | All rights reserved - 133617. You may not reproduce, distribute or exploit the contents in any form without written permission by copyright owner. Copyright infringers may face civil and criminal liability Online Prelims Thematic TEST - 10 ( InsightsIAS Mock Test Series for UPSC Preliminary Exam 2020 )

reflection, resulting in a net warming effect on the climate. Therefore, thinning or removing these clouds would reduce their heat trapping capacity, resulting in a cooling effect on Earth’s climate.

See https://en.wikipedia.org/wiki/Cirrus_cloud_thinning and https://en.wikipedia.org/wiki/Stratosphe ric_aerosol_injection

Q Source: AR: UPSC CSP 2019

21 Social capital and human capital of a nation increases if 1. the number of skilled and educated people increase 2. overall health indicators of the nation improve 3. there is greater harmony and peace in the nation

Which of the statements given above are correct? A. 1 and 2 only B. 2 and 3 only C. 1 and 3 only D. 1, 2 and 3

Your Answer : Correct Answer : D

Answer Justification :

Justification: We’ve already covered human capital in previous tests.

Social capital is a set of shared values that allows individuals to work together in a group to effectively achieve a common purpose.

The idea is generally used to describe how members are able to band together in society to live harmoniously. In business, social capital can contribute to a company's success by building a sense of shared IMAGERUNNERSvalues and mutual respect. 8860450330 Social capital can manipulate people and destroy order as is the case with drug cartels and corporations that team up to drive out the competition.

So, in other words, Social capital broadly refers to those factors of effectively functioning social groups that include such things as interpersonal relationships, a shared sense of identity, a shared understanding, shared norms, shared values, trust, cooperation, and reciprocity.

Q Source: AR: UPSC 2019

22 Internal and Extra Budgetary Resources (IEBR) is essentially

A. Private commercial borrowing that does not create crowding out effect B. Re-capitalization of Public Sector Banks that does not cost the exchequer C. RBI’s Capital Framework money which is due to the government prelims.insightsonindia.com 17 © Insights Active Learning | All rights reserved - 133617. You may not reproduce, distribute or exploit the contents in any form without written permission by copyright owner. Copyright infringers may face civil and criminal liability Online Prelims Thematic TEST - 10 ( InsightsIAS Mock Test Series for UPSC Preliminary Exam 2020 )

D. Part of Central plan constituting resources raised by the PSUs through profits, loans and equity

Your Answer : Correct Answer : D

Answer Justification :

Learning: IEBR is an important part of the Central plan of the Government of India and constitutes the resources raised by the PSUs through profits, loans and equity.

The Union Cabinet has recently given approval to service principal and interest against the Extra Budgetary Resources (EBR) of Rs. 16,300 crore by Union Government for infrastructure spending. The EBR is proposed to finance funds to be raised by Indian Renewable Energy Development Agency (IREDA), Power Finance Corporation (PFC), Inland Waterways Authority of India (IWAI), and National Bank for Agriculture and Rural Development (NABARD). This implies that interest and principal in respect of the EBR of Rs.16,300 crore will be financed by Union Government by making suitable budget provisions. It will supplement the efforts of the Union Government to improve infrastructure spending and also improve revenue-capital mix of expenditure for a more sustainable growth.

Q Source: Based on UPSC papers

23 Consider the following statements. 1. Kalyaana Mandapas were halls used for marriage or other ceremonies in Vijayanagara empire. 2. Open matapas (mandapas) were a feature of some Vijayanagar style temples, such as in Hampi.

Which of the above is/are correct? A. 1 only B. 2 only C.IMAGERUNNERSBoth 1 and 2 8860450330 D. None

Your Answer : Correct Answer : C

Answer Justification :

Justification: Vijayanagara temples are usually surrounded by a strong enclosure. Small shrines consist simply of a garbhagriha (sanctum) and a porch. Medium-sized temples have a garbhagriha, shukanasi (antechamber), a navaranga (antrala) connecting the sanctum and outer mandapa (hall), and a rangamantapa (enclosed pillared hall). Large temples have tall Rayagopuram built with wood, brick and stucco in Chola style.

Kalyaana Mandapas were halls used for marriage or other ceremonies in Vijayanagara empire. See https://en.wikipedia.org/wiki/Vijayanagara_architecture#Temple_structures

prelims.insightsonindia.com 18 © Insights Active Learning | All rights reserved - 133617. You may not reproduce, distribute or exploit the contents in any form without written permission by copyright owner. Copyright infringers may face civil and criminal liability Online Prelims Thematic TEST - 10 ( InsightsIAS Mock Test Series for UPSC Preliminary Exam 2020 )

Statement 2: Pillared open mantapa incorporating Hoysala style "staggered square" layout at Vittala temple in Hampi in the image below:

Q Source: Based on UPSC past year papers

24 Consider the following statements: 1. Saint Nimbarka was a Hindu philosopher who was known for propagating the Vaishnava doctrine of unity in duality. 2. Shaikh Ahmad Sirhindi was a member of the Naqshbandī Sufi order.

Which of the statements given above is/are correct? A. 1 only B. 2 only C. Both 1 and 2 D. Neither 1 nor 2

Your IMAGERUNNERS Answer : 8860450330 Correct Answer : B

Answer Justification :

Learning: Statement 1: Nimbarka, was a Hindu philosopher and commentator, known for propagating the Vaishnava doctrine of bhedabheda dvaitadvaita, duality in unity. According to the Vedic scriptures, he was born in 3096 B.C.E., but modern historical research places him in the thirteenth or fourteenth century. See https://www.newworldencyclopedia.org/entry/Nimbarka

Statement 2: Shaikh Ahmad Sirhindi was born after Saint Kabir.

https://en.wikipedia.org/wiki/Ahmad_Sirhindi

Ahmad al-Fārūqī al-Sirhindī (1564–1624) was an Indian Islamic scholar, a Hanafi jurist, and a member of the Naqshbandī Sufi order. He has been described by some followers as a Mujaddid, meaning a “reviver", for his work in rejuvenating Islam and opposing the newly made religion of prelims.insightsonindia.com 19 © Insights Active Learning | All rights reserved - 133617. You may not reproduce, distribute or exploit the contents in any form without written permission by copyright owner. Copyright infringers may face civil and criminal liability Online Prelims Thematic TEST - 10 ( InsightsIAS Mock Test Series for UPSC Preliminary Exam 2020 )

Din-i Ilahi and other dissident opinions of Mughal emperor Akbar.

Most of the Naqshbandī suborders today, such as the Mujaddidī, Khālidī, Saifī, Tāhirī, Qasimiya and Haqqānī sub-orders, trace their spiritual lineage through Sirhindi.

Q Source: AR: UPSC CSP 2019

25 With reference to the Gandhi’s role in colonial India, consider the following statements: 1. Mahatma Gandhi was instrumental in getting the indentured labour system abolished in the British Empire in the 19th CE. 2. In Lord Chelmsford’s ‘War Conference’, Mahatma Gandhi was not invited.

Which of the statements given above are correct? A. 1 only B. 2 only C. Both 1 and 2 D. None

Your Answer : Correct Answer : D

Answer Justification :

Justification: S1: In his famous autobiography "The Story of My Experiment with Truth", the Mahatma explained that during his three-week stay in Mauritius he had acquainted himself fairly well with local conditions in the colony. This was the primary reason why in 1906, during a brief meeting with Manilal Doctor in London, Gandhi asked him to go to Mauritius. It becomes evident that the plight of the colony's -Mauritians and the Indian indentured labourers preoccupied the apostle of non-violence even several years after leaving our shores.

During the early 1900s (20th CE), Gandhi was instrumental in getting the indentured labour IMAGERUNNERS system abolished in 8860450330the British Empire. In his letter of 1911 to Gokhale, Gandhi stated that Manilal Doctor was proceeding to India to attend an important meeting of the Indian National Congress where he would campaign to get a resolution passed condemning the indentured labour system in all British territories. The Mahatma mentioned that he supported Manilal's initiative who has seen the terrible living and working conditions of the indentured workers in Mauritius.

See https://www.mkgandhi.org/articles/mauritius&mg.htm

S2: During 1917, the World war had entered a critical phase. Britain and France were in a difficult position. The Viceroy of India Lord Chelmsford, invited various Indian leaders to attend a War conference. Gandhi was also invited. He accepted the invitation and went to Delhi Gandhi was not happy that leaders like Tilak or the Ali brothers had not been invited to the conference, so he felt unwilling to attend. After meeting the Viceroy, however, he attended the conference.

The Viceroy was very keen that Gandhi should support the resolution on recruiting. He supported it. Pasting this section from B.R. Nanda’s book:

prelims.insightsonindia.com 20 © Insights Active Learning | All rights reserved - 133617. You may not reproduce, distribute or exploit the contents in any form without written permission by copyright owner. Copyright infringers may face civil and criminal liability Online Prelims Thematic TEST - 10 ( InsightsIAS Mock Test Series for UPSC Preliminary Exam 2020 )

See https://tinyurl.com/yxroyhnd

Q Source: AR: UPSC CSP 2019

26 The Sarkaria Commission recommended that the Governor to be appointed 1. should be an eminent person from inside the state 2. should be a detached figure without intense political links 3. should not have taken part in politics in recent past and should not be a member of the ruling party

Which of the above is/are correct? A. 1 and 2 only B. 2 and 3 only C. 1 and 3 only D. 1, 2 and 3

Your Answer : Correct Answer : B

Answer Justification :

Justification: Recommendations on Appointment of Governor :

1. should be an eminent person; 2. must be a person from outside the State; 3. not have participated in active politics at least for some time before his appointment; 4. he should be a detached person and not too intimately connected with the local politics of the State; 5. he should be appointed in consultation with the Chief Minister of the State, Vice-President of India and the Speaker of the Lok Sabha; 6. His tenure of office must be guaranteed and should not be disturbed except for extremely compelling reasons and if any action is to be taken against him he must be given a reasonable IMAGERUNNERSopportunity for showing 8860450330 cause against the grounds on which he is sought to be removed. In case of such termination or resignation of the Governor, the Government should lay before both the Houses of Parliament a statement explaining the circumstances leading to such removal or resignation, as the case may be; 7. After demitting his office, the person appointed as Governor should not be eligible for any other appointment or office of profit under the Union or a State Government except for a second term as Governor or election as Vice-President or President of India, as the case may be; and 8. At the end of his tenure, reasonable post-retirement benefits should be provided.

Q Source: AR: UPSC 2019

27 This national park is famous for its alpine meadows and it wholly lies in temperate alpine zone. It is

A. Neora Valley National Park B. Valley of Flowers National Park prelims.insightsonindia.com 21 © Insights Active Learning | All rights reserved - 133617. You may not reproduce, distribute or exploit the contents in any form without written permission by copyright owner. Copyright infringers may face civil and criminal liability Online Prelims Thematic TEST - 10 ( InsightsIAS Mock Test Series for UPSC Preliminary Exam 2020 )

C. Manas National Park D. Namdapha National Park

Your Answer : Correct Answer : B

Answer Justification :

Justification: Valley of flowers is famous for its alpine meadows. It wholly lies in temperate alpine zone. The valley has three sub-alpine between 3,200m and 3,500m which is the limit for trees, lower alpine between 3,500m and 3,700m, and higher alpine above 3,700m.

This richly diverse area is also home to rare and endangered animals, including the Asiatic black bear, snow leopard, musk deer, brown bear, red fox and blue sheep. Birds found in the park include Himalayan monal pheasant and other high altitude birds.

At 3352 to 3658 meters above sea level, the gentle landscape of the Valley of Flowers National Park complements the rugged mountain wilderness of Nanda Devi National Park to the east.

See https://en.wikipedia.org/wiki/Valley_of_Flowers_National_Park#Vegetation

Q Source: Slightly modified: UPSC CSP 2019

28 In addition to the minor minerals specified in the Mines and Minerals (Development and Regulation) Act, 1957 (MMDR) Act, the Central Government has declared the following minerals as minor minerals: 1. Lignite 2. Brick-earth 3. Gold 4. Marble

Which of the statements given above is / are correct? A.IMAGERUNNERS1 and 3 only 8860450330 B. 2 and 4 only C. 3 only D. 1, 2, 3 and 4

Your Answer : Correct Answer : B

Answer Justification :

Justification: minerals are those specified in the first schedule appended in the Mines and Minerals (Development and Regulation) Act, 1957 (MMDR Act 1957) and the common major minerals are Lignite, Coal, Uranium, iron ore, gold etc. It may be noted that there is no official definition for “major minerals” in the MMDR Act. Hence, whatever is not declared as a “minor mineral” may be treated as the major mineral.

prelims.insightsonindia.com 22 © Insights Active Learning | All rights reserved - 133617. You may not reproduce, distribute or exploit the contents in any form without written permission by copyright owner. Copyright infringers may face civil and criminal liability Online Prelims Thematic TEST - 10 ( InsightsIAS Mock Test Series for UPSC Preliminary Exam 2020 )

According to section 3(e) of the Mines and Minerals (Development and Regulation) Act, 1957 “Minor Minerals” means building stones, gravel, ordinary clay, ordinary sand other than sand used for prescribed purposes.

India produces as many as 88 minerals which include 4 fuel minerals, 3 atomic minerals, 26 metallic & non-metallic minerals and 55 minor minerals (including building and other materials and the recently notified 31 additional minerals).

The central government has the power to notify “minor minerals” under section 3 (e) of the MMDR Act, 1957. On the other hand, as per Section 15 of the MMDR Act, 1957 State Governments have complete powers for making Rules for grant of concessions in respect of extraction of minor minerals and levy and collection of royalty on minor minerals.

In addition to the minor minerals specified in Section 3(e) of the MMDR Act, the Central Government has declared the following minerals as minor minerals:

boulder, shingle, chalcedony pebbles used for ball mill purposes only, lime shell, kankar and limestone used in kilns for manufacture of lime used as building material, murrum, brick-earth, fuller’s earth, bentonite, road metal, reh-matti, slate and shale when used for building material, marble, stone used for making household utensils, quartzite and sandstone when used for purposes of building or for making road metal and household utensils, IMAGERUNNERSsaltpeter and 8860450330 ordinary earth (used or filling or leveling purposes in construction or embankments, roads, railways, building).

The power to frame policy and legislation relating to minor minerals is entirely delegated to the State Governments while policy and legislation relating to the major minerals are dealt by the Ministry of Mines under Union /Central Government. So, S2 is incorrect and S3 is correct.

Q Source: AR: UPSC 2019

29 Consider the following statements. 1. Loans and Advances form liabilities of a commercial bank. 2. Deposits are an asset for a commercial bank.

Which of the above is/are correct? A. 1 only prelims.insightsonindia.com 23 © Insights Active Learning | All rights reserved - 133617. You may not reproduce, distribute or exploit the contents in any form without written permission by copyright owner. Copyright infringers may face civil and criminal liability Online Prelims Thematic TEST - 10 ( InsightsIAS Mock Test Series for UPSC Preliminary Exam 2020 )

B. 2 only C. Both 1 and 2 D. None

Your Answer : Correct Answer : D

Answer Justification :

Justification: Deposits are a liability on a commercial bank, since it must return its lenders the amount it owes them.

Loans and Advances form assets of the bank. Also, Investments form the second largest component in the assets side of banks’ balance sheets after loans and advances as per a recent RBI report, see https://rbi.org.in/scripts/PublicationsView.aspx?Id=18743 (sec 2.4)

So, the assets of the commercial bank in India are Cash in hand, Investments, Loans, Advances, Bills discounted and purchased, while the liabilities are Capital and Reserves, Deposits, Borrowings, and other liabilities.

Q Source: AR: UPSC 2019

30 Consider the following about the “Hind Mazdoor Sabha”. 1. It was established pre-independence. 2. It absorbed the Indian Federation of Labour and the Hind Mazdoor Panchayat.

Which of the above is/are correct? A. 1 only B. 2 only C. Both 1 and 2 D. None IMAGERUNNERS 8860450330 Your Answer : Correct Answer : B

Answer Justification :

Justification: The HMS was founded in Howrah in 1948 by socialists Forward Bloc followers and independent unionists which included Basawon Singh (Sinha), Ashok Mehta, R.S. Ruikar, Maniben Kara, Shibnath Banerjee, R.A. Khedgikar, T.S. Ramanujam, V.S. Mathur, G.G. Mehta. R.S. Ruikar was elected president and Ashok Mehta general secretary.

HMS absorbed the Indian Federation of Labour (founded by Manabendra Nath Roy) and the Hind Mazdoor Panchayat, which was formed in 1948 by socialists leaving the increasingly communist dominated AITUC.

Q Source: Based on UPSC papers

prelims.insightsonindia.com 24 © Insights Active Learning | All rights reserved - 133617. You may not reproduce, distribute or exploit the contents in any form without written permission by copyright owner. Copyright infringers may face civil and criminal liability Online Prelims Thematic TEST - 10 ( InsightsIAS Mock Test Series for UPSC Preliminary Exam 2020 )

31 Consider the following about the “Sthanakvasi” religious . 1. It is a sect of Digambara Jainism. 2. It believes that idol worship is not essential in the path of soul purification. 3. It does not believe in the attainment of Moksha or Nirvana.

Which of the above is/are correct? A. 1 and 2 only B. 3 only C. 2 only D. 1, 2 and 3

Your Answer : Correct Answer : C

Answer Justification :

Justification: Sthānakavāsī is a sect of Śvētāmbara Jainism founded by a merchant named Lavaji in 1653 AD. It believes that idol worship is not essential in the path of soul purification and attainment of Nirvana/Moksha.

The sect is essentially a reformation of the one founded on teachings of Lonka, a fifteenth-century Jain reformer. Sthānakavāsins accept thirty-two of the Jain Agamas, the Śvētāmbara canon. Śvētāmbarins who are not Sthānakavāsins are mostly part the Murtipujaka sect

See https://en.wikipedia.org/wiki/Sth%C4%81nakav%C4%81s%C4%AB

Q Source: AR: UPSC CSP 2018

32 Consider the following statements. 1. The Kurvey Committee of 1955 led to the establishment of Khadi Village and Industries Commission (KVIC). 2. The IMAGERUNNERS Zaman Committee of 1959 8860450330 advocated the decentralization of Khadi work, recognition of large certified institutions and formation of co-operatives. 3. The Gyanchand Committee of 1962 was appointed to evaluate Khadi sales in the country.

Which of the above is/are correct? A. 1 and 2 only B. 2 and 3 only C. 1, 2 and 3 D. 1 only

Your Answer : Correct Answer : C

Answer Justification :

Background: The first plan had adopted a complete 'Gandhian' perspective in development of KVIC as it was decided to be developed 'with processing of local raw material for the local market with prelims.insightsonindia.com 25 © Insights Active Learning | All rights reserved - 133617. You may not reproduce, distribute or exploit the contents in any form without written permission by copyright owner. Copyright infringers may face civil and criminal liability Online Prelims Thematic TEST - 10 ( InsightsIAS Mock Test Series for UPSC Preliminary Exam 2020 )

simple techniques (1951).

As an appropriate method of protection a "Common Minimum Programme" was formulated, which was mainly related to reservation of production, restriction on capacity expansion and continuation of research. A multi-institutional approach was developed by establishing a separate institution like the KVIC Board, Hand-loom Board, Handicraft Board and Small Scale Industries Board for their development.

The Second Five Year Plan gave a very strategic place to village industries to generate marketable surplus as consumer goods to support heavy industry development without inflation and also gave a task to liquidate unemployment as quickly as possible. The basic approach for the KVIC was worked out by the panel of economists appropriate to the development of these sectors.

Justification: The Kurvey Committee of 1955 led to the establishment of KVIC and it also suggested distribution of 2-5 million ambar charkhas- technologically improved hand spinning equipment. The Zaman Committee advocated the decentralization of Khadi work, recognition of large certified institutions and formation of co-operatives (1959).

The Gyanchand Committee appointed to evaluate Khadi, pointed out the vicious cycle of low output, low wages and even falling wages as the central problem. It advocated that the yarn production through the traditional charkha to provide relief to distressed persons should be separated from the economic problem for Khadi production as an employment generation activity.

The Nathu Committee in 1962 recommended that the policy of production and sales should be reoriented to effect at least 40% of sales within the district and 80% within the state and export to other states should not exceed 20%.

The Ashok Mehta Committee on KVIC in 1968 attempted to evolve a fresh approach to development based on the three basic components of producing salable articles, providing employment to people in backward areas, tribal and inaccessible areas, famine and drought stricken areas and also the backward and less privileged section of the population and to create self-reliance and community spirit among rural people.

Q Source:IMAGERUNNERS Based on CSP papers 8860450330 + https://www.mkgandhi.org/articles/G%20and%20the%2021st%20century.htm

33 He is considered to be a in Mahayana Buddhist tradition, as the embodiment of infinite compassion. Tibetan tradition calls him Cherezig and regards him as a Buddha, and the Dalai to be his incarnation. He is

A. Avalokiteshvara B. Lokesvara C. Maitreya D. Padmapani

Your Answer : Correct Answer : A

prelims.insightsonindia.com 26 © Insights Active Learning | All rights reserved - 133617. You may not reproduce, distribute or exploit the contents in any form without written permission by copyright owner. Copyright infringers may face civil and criminal liability Online Prelims Thematic TEST - 10 ( InsightsIAS Mock Test Series for UPSC Preliminary Exam 2020 )

Answer Justification :

Justification: Avalokiteshvara is a high-level bodhisattva (“enlightenment being”) in Mahayana Buddhist tradition, expressing the important element of compassion in the Dharma teachings. His name means “Lord who looks down [and hears the cries of the world]”. He was popular in India until the twelfth century, when Muslim invaders expelled Buddhists.

Tibetan Vajrayana tradition regards him as a Buddha. They developed many variations on his imagery, some with many faces or many arms. They see the ultimate source of his tradition as the universal manifestation of compassion itself. Some see the as an incarnation of Avalokiteshvara, also named Chenrezig. His anthropomorphized images are useful ways of conveying his teachings to humans. Some Tibetan traditions tell the story that the goddess Tara came from one of Chenrezig’stears that fell and turned into a lake, and a lotus grew up and revealed Tara. Another account portrays her emerging from his heart. Chinese Buddhists see him in the.

On the other hand, Maitreya is regarded as a future Buddha of this world in Buddhist eschatology. In some Buddhist literature, such as the Amitabha Sutra and the Lotus Sutra, he is referred to as Ajita.

According to Buddhist tradition, Maitreya is a bodhisattva who will appear on Earth in the future, achieve complete enlightenment, and teach the pure dharma.

Q Source: AR: UPSC CSP 2018

34 Consider the following statements. 1. Molds are microscopic fungi that live on plant or animal matter. 2. Mycotoxins are poisonous substances produced by certain molds found primarily in grain and nut crops. 3. Food and Agriculture Organization (FAO) estimates that about one-fourth of the world's food crops are affected by mycotoxins.

Which IMAGERUNNERS of the above is/are correct? 8860450330 A. 1 only B. 2 and 3 only C. 1, 2 and 3 D. 1 and 3 only

Your Answer : Correct Answer : C

Answer Justification :

Justification: Molds are microscopic fungi that live on plant or animal matter.

Mycotoxins are poisonous substances produced by certain molds found primarily in grain and nut crops, but are also known to be on celery, grape juice, apples, and other produce. Mycotoxins can penetrate into the parts of food that are not visibly mouldy as well. It is prelims.insightsonindia.com 27 © Insights Active Learning | All rights reserved - 133617. You may not reproduce, distribute or exploit the contents in any form without written permission by copyright owner. Copyright infringers may face civil and criminal liability Online Prelims Thematic TEST - 10 ( InsightsIAS Mock Test Series for UPSC Preliminary Exam 2020 )

therefore necessary to throw away all of the food if any part of it is mouldy. They are also notoriously difficult to destroy as they are stable to both heat and chemicals. S3 is correct, since FAO of the UN notes that mycotoxins and in particular alfatoxins, are harming nearly 25% of world’s food crops.

Learning: Aflatoxin is (poison) any of a family of mycotoxins, produced by molds of the aspergillus genus.

Aflatoxins have been associated with various diseases, such as aflatoxicosis in livestock, domestic animals, and humans throughout the world. Patulin is a mycotoxin produced by a variety of molds, in particular, Aspergillus and Penicillium and Byssochlamys. Most commonly found in rotting apples, in general the amount of patulin in products is viewed as a measure of the quality of the apples used in production. Aspergillus is used in the commercial production of citric acid.

Q Source: Based on CAPF and CSP papers

35 Consider the following about all-India Household Consumer Expenditure Survey (CES). 1. CES was earlier conducted by the National Sample Survey Office (NSSO). 2. CES is conducted only in rural areas, since demand in urban areas can be estimated with other indicators. 3. Data from CES is useful in deciding the base years of GDP growth data and other macroeconomic indicators.

Which of the above is/are correct? A. 1 only B. 1 and 3 only C. 2 and 3 only D. 1, 2 and 3

Your IMAGERUNNERS Answer : 8860450330 Correct Answer : B

Answer Justification :

Learning: The CES is traditionally a quinquennial (recurring every five years) survey conducted by the government’s National Sample Survey Office (NSSO) that is designed to collect information on the consumption spending patterns of households across the country, both urban and rural.

The data gathered in this exercise reveals the average expenditure on goods (food and non-food) and services and helps generate estimates of household Monthly Per Capita Consumer Expenditure (MPCE) as well as the distribution of households and persons over the MPCE classes.

The estimates of monthly per capita consumption spending are vital in gauging the demand dynamics of the economy as well as for understanding the shifting priorities in terms of baskets of goods and services, and in assessing living standards and growth trends across multiple strata. From helping policymakers spot and address possible structural anomalies that may cause demand prelims.insightsonindia.com 28 © Insights Active Learning | All rights reserved - 133617. You may not reproduce, distribute or exploit the contents in any form without written permission by copyright owner. Copyright infringers may face civil and criminal liability Online Prelims Thematic TEST - 10 ( InsightsIAS Mock Test Series for UPSC Preliminary Exam 2020 )

to shift in a particular manner in a specific socio-economic or regional cohort of the population, to providing pointers to producers of goods and providers of services, the CES is an invaluable analytical as well as forecasting tool.

It is, in fact, used by the government in rebasing the GDP and other macro-economic indicators.

See https://www.thehindu.com/business/Economy/what-is-consumer-expenditure-survey-and-why-was-its -2017-2018-data-withheld/article30063708.ece

Q Source: AR: UPSC CSP papers

36 Consider the following statements: As per the Constitution of India 1. the executive powers of the State extends to borrowing within the territory of Indian but only upon the security of the Consolidated Fund of India within certain limits as stipulated by the Parliament 2. it is mandatory for a State to take the Central Government’s consent for raising any loan if the former owes any outstanding liabilities to the letter.

Which of the statements given above is/are correct? A. 1 only B. 2 only C. Both 1 and 2 D. None

Your Answer : Correct Answer : B

Answer Justification :

Justification: IMAGERUNNERS In terms of Article 8860450330 293(1) of the Constitution of the India, the State governments can give guarantees within such limits as fixed by the legislature of the concerned State. The constitutional provision relating to raising of loans and issuing of guarantees are summarised below :

1. Subject to the provisions of this Article, the executive powers of the State extends to borrowing within the territory of Indian upon the security of the Consolidated Fund of the State within such limits, if any, as may from time to time be fixed by the Legislature of such State by law and to the giving of guarantees within such limits, if any, as may be so fixed. 2. The Government of India may subject to such conditions as may be laid down by or under any law made by Parliament , make loans to any State or, so long as any limits fixed under Article 292 are not exceeded, give guarantees in respect of loans raised by any State, and any sums required for the purpose of making such loans shall be charged on the Consolidated Fund of India. 3. A State may not without the consent of the Government of India raise any loan if there is still outstanding any part of a loan which has been made to the State by the Government of India or by its predecessor Government , or in respect of which guarantee has been given by the prelims.insightsonindia.com 29 © Insights Active Learning | All rights reserved - 133617. You may not reproduce, distribute or exploit the contents in any form without written permission by copyright owner. Copyright infringers may face civil and criminal liability Online Prelims Thematic TEST - 10 ( InsightsIAS Mock Test Series for UPSC Preliminary Exam 2020 )

Government of India or its predecessor Government. 4. A consent under Clause (3) may be granted subject to such conditions if any as the Government of India may think fit to impose.

See https://www.rbi.org.in/Scripts/PublicationReportDetails.aspx?ID=17

Q Source: AR: UPSC CSP papers

37 Which of these tribes found in the Eastern Himalayan region practice Theravada Buddhism? 1. Rabhas 2. Hajongs 3. Khamti 4. Chakmas

Select the correct answer using the codes below. A. 1, 2 and 3 only B. 3 and 4 only C. 1 and 2 only D. 1, 2, 3 and 4

Your Answer : Correct Answer : B

Answer Justification :

Justification: Statement 1: Rabha is a Scheduled Tribe community of Assam, Meghalaya and West Bengal. The language/dialect spoken by the Rabha people is also of the same name.

The Rabhas belong to the Indo Mongoloid group of people and have similarities with other members of Koch group such as Garos.

Statement 2: They are one of the largest ethnicities in Meghalaya. Hajong people are spread out across IMAGERUNNERS North East India West Bengal8860450330 and Bangladesh.

Hajongs are predominantly rice farmers. Hajong have the status of a Scheduled Tribe in India.

Statement 3: They can be found in northwestern Burma as well as Lohit district of Arunachal Pradesh in India. Smaller numbers can be found in parts of Assam.

The Tai-Khamti are followers of Theravada Buddhism.

Statement 4: Chakmas are spread across Bangladesh and parts of northeastern India, western Burma etc.

They have their own language, customs and culture, and profess Theravada Buddhism.

Q Source: Based on CSP papers

prelims.insightsonindia.com 30 © Insights Active Learning | All rights reserved - 133617. You may not reproduce, distribute or exploit the contents in any form without written permission by copyright owner. Copyright infringers may face civil and criminal liability Online Prelims Thematic TEST - 10 ( InsightsIAS Mock Test Series for UPSC Preliminary Exam 2020 )

38 Which of the following disease(s) has/have been globally eradicated? 1. Smallpox 2. Rinderpest 3. Polio

Select the correct answer using the codes below. A. 1, 2 and 3 B. 2 only C. 1 and 2 only D. 3 only

Your Answer : Correct Answer : C

Answer Justification :

Justification: Diseases that are considered eradicable today are: polio, Guinea worm disease, lymphatic filariasis, cysticercosis, measles, mumps and rubella

Eradicable diseases usually need to meet the following criteria: it’s an infectious disease, humans are the major host of for the disease, effective vaccines or treatments are available for the disease, and there is political and financial support for the eradication efforts

The world has eradicated two diseases: Smallpox and Rinderpest

Efforts to eradicate or eliminate additional diseases from parts of the world such as malaria, trachoma, river blindness and yaws are underway.

As at 2020, wild poliovirus type 1 affects two countries: Pakistan and Afghanistan.

About India:

IMAGERUNNERSIndia has become the first 8860450330 country in the world to get the Yaws-free certificate by the WHO. This is the third major momentous public health feat for India, which was declared polio-free two years ago, elimination of maternal and neonatal tetanus early this year and now becoming free from Yaws, a chronic bacterial infection caused by Treponema pallidum subsp Pertenue. India has officially declared itself free from highly contagious Avian Influenza (H5N1) or bird flu and has notified the same to the World Organization for Animal Health (OIE). Smallpox was declared by WHO to be eradicated in 1979 from the World.

Q Source: Based on UPSC CSP papers

39 Consider the following statements. 1. Montreal Protocol 1987 targets phasing out production of numerous Ozone Depleting Substances (ODSs). 2. Vienna Convention for the Protection of the Ozone Layer includes legally binding reduction goals for the use of Chlorofluorocarbons (CFCs). 3. Methyl Chloroform and Methyl Bromide are ODSs that have been phased-out globally. prelims.insightsonindia.com 31 © Insights Active Learning | All rights reserved - 133617. You may not reproduce, distribute or exploit the contents in any form without written permission by copyright owner. Copyright infringers may face civil and criminal liability Online Prelims Thematic TEST - 10 ( InsightsIAS Mock Test Series for UPSC Preliminary Exam 2020 )

Select the correct answer using the codes below. A. 1 and 3 only B. 1 and 2 only C. 3 only D. 1, 2 and 3

Your Answer : Correct Answer : A

Answer Justification :

Justification: Montreal Protocol on Substances that Deplete the Ozone Layer

It is an international treaty designed to protect the ozone layer by phasing out production of numerous Ozone Depleting Substances (ODSs) that are responsible for ozone depletion. It was agreed on 26 August 1987 in Montreal, Canada and entered into force on 26 August 1989. It was followed by a first meeting in Helsinki, May 1989. Due to its universality, this international agreement has helped in recovering the ozone hole in Antarctica. Under it production and consumption of key ODSs like chlorofluorocarbon (CFCs), Methyl Chloroform, CTC halons and Methyl Bromide have been phased-out globally.

However, it does not include legally binding reduction goals for the use of CFCs, the main chemical agents causing ozone depletion.

The Vienna Convention for the Protection of the Ozone Layer is a multilateral environmental agreement signed in 1985 that provided frameworks for international reductions in the production of chlorofluorocarbons due to their contribution to the destruction of the ozone layer. It is not legally binding.

Q Source: Based on UPSC CSP papers IMAGERUNNERS 8860450330 40 Consider the following statements about a protected area. 1. It is the largest protected area in the Eastern Himalaya biodiversity hotspot and harbours the northernmost lowland evergreen rainforests in the world. 2. The habitat changes with increasing altitude from tropical moist forests to Montane forests, temperate forests and at the higher elevations to Alpine meadows and perennial snow.

The national park referred above is? A. Namdapha National Park B. Dachigam National park C. Nokrek National park D. Khangchendzonga National Park

Your Answer : Correct Answer : A

prelims.insightsonindia.com 32 © Insights Active Learning | All rights reserved - 133617. You may not reproduce, distribute or exploit the contents in any form without written permission by copyright owner. Copyright infringers may face civil and criminal liability Online Prelims Thematic TEST - 10 ( InsightsIAS Mock Test Series for UPSC Preliminary Exam 2020 )

Answer Justification :

Learning: The park is located in Changlang district of the North-eastern state of Arunachal Pradesh, near its border with Myanmar.

The park has extensive bamboo forests and secondary forests in addition to the primary forests.

Because of many different vegetation zones, the park is home to a great diversity of mammal species. snow leopards, clouded leopards, large Indian civet, barking deer etc.

Q Source: Based on CSP: http://www.worldwildlife.org/places/eastern-himalayas

41 This foreign traveller is best known for the discovery/purchase of a Blue Diamond that was most likely of Indian origin?

A. Marco Polo B. Megasthenes C. Jean Baptiste Tavernier D. Nicolo Conti

Your Answer : Correct Answer : C

Answer Justification :

Justification: Jean-Baptiste Tavernier (1605 – 1689) was a 17th-century French gem merchant and traveller Tavernier, a private individual and merchant traveling at his own expense, covered, by his own account, 60,000 leagues (120,000 miles in making six voyages to Persia and India between the years 1630 and 1668.

Tavernier is best known for his discovery or purchase of the 116-carat Tavernier Blue diamond, in 1666. The diamond was certainly Indian in origin and likely sourced by Tavernier in 1666[4] at the Kollur IMAGERUNNERS mine in Guntur district 8860450330of Andhra Pradesh.

Tavernier sold it to Louis XIV of France.

Q Source: AR: UPSC CSP 2018

42 The Roaring Forties are 1. Strong easterly winds found in the Southern Hemisphere 2. generally found between the latitudes of 40 and 50 degrees

Which of the above is/are correct? A. 1 only B. 2 only C. Both 1 and 2 D. None

prelims.insightsonindia.com 33 © Insights Active Learning | All rights reserved - 133617. You may not reproduce, distribute or exploit the contents in any form without written permission by copyright owner. Copyright infringers may face civil and criminal liability Online Prelims Thematic TEST - 10 ( InsightsIAS Mock Test Series for UPSC Preliminary Exam 2020 )

Your Answer : Correct Answer : B

Answer Justification :

Learning: They are generally found between the latitudes of 40 and 50 degrees.

The strong west-to-east air (Westerly) currents are caused by the combination of air being displaced from the Equator towards the South Pole and the Earth's rotation, and there are few landmasses to serve as windbreaks. The Roaring Forties were a major aid to ships sailing the Brouwer Route from Europe to the East Indies or Australasia during the Age of Sail, and in modern usage are favoured by yachtsmen on round-the-world voyages and competitions. The boundaries of the Roaring Forties are not consistent, and shift north or south depending on the season. Similar but stronger conditions occur in more southerly latitudes and are referred to as the Furious Fifties and Shrieking or Screaming Sixties.

Q Source: Based on CAPF papers

43 Lotus Sutra and Heart Sutra are religious texts ascribed to

A. Saktism B. Vaishnavism C. Buddhism D. Jainism

Your Answer : Correct Answer : C

Answer Justification : IMAGERUNNERS 8860450330 Learning: The Lotus Sutra is widely regarded as one of the most important and influential sutras, or sacred scriptures, of Buddhism. In it, Shakyamuni expounds the ultimate truth of life to which he was enlightened. The sutra’s key message is that , the supreme state of life characterized by boundless compassion, wisdom and courage, is inherent within every person without distinction of gender, ethnicity, social standing or intellectual ability.

The Heart Sutra is the most widely known sutra of the Mahayana tradition of . It is part of the Prajnaparamita Sutras, which is a collection of about 40 sutras composed between 100 BCE and 500 CE. The Heart Sutra is a presentation of profound wisdom on the nature of emptiness or interdependent origination.

Q Source: Based on CSP papers

44 Consider the following about CERN or The European Organization for Nuclear Research. 1. CERN investigates the basic constituents of matter or fundamental particles. prelims.insightsonindia.com 34 © Insights Active Learning | All rights reserved - 133617. You may not reproduce, distribute or exploit the contents in any form without written permission by copyright owner. Copyright infringers may face civil and criminal liability Online Prelims Thematic TEST - 10 ( InsightsIAS Mock Test Series for UPSC Preliminary Exam 2020 )

2. At CERN, Subatomic particles are made to collide together at close to the speed of light. 3. India is a member state of CERN.

Select the correct answer using the codes below. A. 1 and 2 only B. 2 and 3 only C. 1 and 3 only D. 3 only

Your Answer : Correct Answer : A

Answer Justification :

Justification: Physicists and engineers at CERN use the world's largest and most complex scientific instruments to study the basic constituents of matter – fundamental particles. Subatomic particles are made to collide together at close to the speed of light. The process gives us clues about how the particles interact, and provides insights into the fundamental laws of nature. We want to advance the boundaries of human knowledge by delving into the smallest building blocks of our universe.

The instruments used at CERN are purpose-built particle accelerators and detectors. Accelerators boost beams of particles to high energies before the beams are made to collide with each other or with stationary targets. Detectors observe and record the results of these collisions. Founded in 1954, the CERN laboratory sits astride the Franco-Swiss border near Geneva. It was one of Europe's first joint ventures and now has 23 member states.

Statement 3: Today CERN has 23 Member States: Austria, Belgium, Bulgaria, Czech Republic, Denmark, Finland, France, Germany, Greece, Hungary, Israel, Italy, Netherlands, Norway, Poland, Portugal, Romania, Serbia, Slovakia, Spain, Sweden, Switzerland and United Kingdom.Cyprus and Slovenia are Associate Member States in the pre-stage to Membership. Croatia, India, Lithuania, Pakistan, Turkey and Ukraine are Associate Member States. IMAGERUNNERS 8860450330 Member States have special duties and privileges. They make a contribution to the capital and operating costs of CERN’s programmes, and are represented in the Council, responsible for all important decisions about the Organization and its activities.

Q Source: AR: UPSC CAPF papers

45 Consider the following statements. 1. The National Sample Survey Office (NSSO) and the Central Statistics Office have merged into the National Statistical Office (NSO). 2. The NSO is headed by the Vice-chairperson, NITI Aayog.

Which of the above is/are correct? A. 1 only B. 2 only C. Both 1 and 2 prelims.insightsonindia.com 35 © Insights Active Learning | All rights reserved - 133617. You may not reproduce, distribute or exploit the contents in any form without written permission by copyright owner. Copyright infringers may face civil and criminal liability Online Prelims Thematic TEST - 10 ( InsightsIAS Mock Test Series for UPSC Preliminary Exam 2020 )

D. None

Your Answer : Correct Answer : A

Answer Justification :

Learning: On May 23, 2019, the government announced that the National Sample Survey Office (NSSO) will be merged with the Central Statistics Office to form the National Statistical Office (NSO).

Many believe that this move will undermine the autonomy of the NSSO which has been at the centre of various public controversies over data on economic growth and unemployment.

The NSO will be headed by the secretary of the Ministry of Statistics and Programme Implementation (MOSPI). This is in contrast to the original plan proposed by experts to merge various statistical bodies such as the NSSO and others to create a unified statistics body that is accountable to Parliament, rather than the government.

The National Statistical Office is the state statistical agency set up by an Act of the Parliament, Statistical Services Act (Chapter 386) 1980, responsible for collecting, compiling and disseminating official statistical information on a regular and timely basis.

See http://www.nso.gov.pg/index.php/8-home/1-national-statistical-office

https://www.thehindu.com/news/national/what-does-the-merger-of-national-sample-survey-office-and -central-statistics-office-entail/article27401039.ece

Q Source: AR: UPSC CSP papers

46 Which of the following reasons explain as to why the population of Syrus crane has declined in recent years? 1. Wetlands IMAGERUNNERS have been lost and habitat8860450330 land has been converted for agriculture. 2. These birds have often collided with power lines leading to loss of life. 3. Cranes have ingested pesticides while foraging in the crop fields leading to fatality.

Select the correct answer using the codes below. A. 1 and 2 only B. 3 only C. 1 only D. 1, 2 and 3

Your Answer : Correct Answer : D

Answer Justification :

Justification: The Sarus crane is the tallest flying bird in the world. Nests are constructed on prelims.insightsonindia.com 36 © Insights Active Learning | All rights reserved - 133617. You may not reproduce, distribute or exploit the contents in any form without written permission by copyright owner. Copyright infringers may face civil and criminal liability Online Prelims Thematic TEST - 10 ( InsightsIAS Mock Test Series for UPSC Preliminary Exam 2020 )

water in natural wetlands or in flooded paddy fields.

Statement 1: The main threat to the Sarus crane in India is habitat loss and degradation due to draining the wetland and conversion of land for agriculture. The landscape of its historic range is rapidly changing due to construction of highways, housing colonies, roads, and railway lines.

Statement 2 and 3: More recently, many deaths have been recorded due to collision with power lines. Also, due to the increase in agricultural land, Sarus cranes are left with no choice but to forage in these fields, and as a result ingest pesticides, which lead to poisoning.

WWF-India has provided technical support for the restoration and management of key wetlands

King was also called as Piyadassis

47 Who among the following was called as “Piyadassis” in Ancient India?

A. Chroniclers who wrote genealogies B. The Monarch was called as Piyadassis C. Spies of the military department D. Traders who provided services to the mid-town

Your Answer : Correct Answer : B

Answer Justification :

Learning: James Prinsep, an officer in the mint of the East India Company, deciphered Brahmi and Kharosthi, two scripts used in the earliest inscriptions and coins.

He found that most of these mentioned a king referred to as Piyadassi – meaning “pleasant to behold”; there were a few inscriptions which also referred to the king as Asoka, one of the most famous rulers known from . IMAGERUNNERSThis gave a new direction 8860450330 to investigations into early Indian political history as European and Indian scholars used inscriptions and texts composed in a variety of languages to reconstruct the lineages of major dynasties. Devanampiya, often translated as “beloved of the gods” was another term attributed to kings.

Q Source: Based on CAPF papers

48 Consider the following statements. 1. Gandhara and Mathura School of Art flourished during the reign of Kushanas. 2. Mathura school used spotted red sandstone. 3. Mathura school imaged only secular figures.

Which of the above is/are correct? A. 1 and 2 only B. 2 and 3 only prelims.insightsonindia.com 37 © Insights Active Learning | All rights reserved - 133617. You may not reproduce, distribute or exploit the contents in any form without written permission by copyright owner. Copyright infringers may face civil and criminal liability Online Prelims Thematic TEST - 10 ( InsightsIAS Mock Test Series for UPSC Preliminary Exam 2020 )

C. 1, 2 and 3 D. 1 only

Your Answer : Correct Answer : A

Answer Justification :

Learning: Gandhara and Mathura School of Art flourished mainly during reign of Kushana emperor Kanishka during the first century AD. Mathura School had developed indigenously, as one of its distinguishing features.

The Mathura school images include those of Buddha, Bodhisattvas, Vishnu, Shiva, Yakshas, Yakshinis, Jinas etc. representing its vitality and assimilative character as a result of the religious zeal of Brahmanism, Jainism and Buddhism.

Jina Image and Indigenous style of Buddha’s image was a remarkable feature of Mathura art.

Q Source: Based on CAPF papers

49 Consider the following statements. General Anti Avoidance Rules (GAAR) provisions are targeted at 1. Curbing speculative activity in real estate markets 2. Arrangements or transactions made specifically to avoid taxes

Which of the above is/are correct? A. 1 only B. 2 only C. Both 1 and 2 D. None

Your Answer : Correct IMAGERUNNERS Answer : B 8860450330

Answer Justification :

Justification: GAAR provisions are targeted at arrangement or transactions made specifically to avoid taxes.

Evolution: In 2007, Vodafone entered the Indian market by buying Hutchison Essar. The deal took place in Cayman Islands. The Indian government claimed over US$2 billion were lost in taxes.

Later, a notice was sent to Vodafone. Vodafone claimed that the transaction was not taxable as it was between two foreign firms. The government claimed that the deal was taxable as the underlying assets involved were located in India. Then came the provisions of GAAR in India.

Provisions: The regulation allows tax officials to deny tax benefits, if a deal is found without any commercial purpose other than tax avoidance. prelims.insightsonindia.com 38 © Insights Active Learning | All rights reserved - 133617. You may not reproduce, distribute or exploit the contents in any form without written permission by copyright owner. Copyright infringers may face civil and criminal liability Online Prelims Thematic TEST - 10 ( InsightsIAS Mock Test Series for UPSC Preliminary Exam 2020 )

It allows tax officials to target participatory notes. Under GAAR, the investor has to prove that the participatory note was not set to avoid taxes. It also allows officials to deny double taxation avoidance benefits, if deals made in tax havens like Mauritius were found to be avoiding taxes

Q Source: Based on CAPF papers

50 Consider the following statements. 1. Gandhi insisted on "Primitive methods" of production in the khadi village industry. 2. Gandhi did not advocate selling any surplus produce derived from Khadi. 3. Gandhi was against Mechanization since it replaced labour.

Which of the above is/are correct? A. 1 and 2 only B. 2 and 3 only C. 1, 2 and 3 D. 1 and 3 only

Your Answer : Correct Answer : D

Answer Justification :

Learning: Gandhi had not perhaps conceptualized the Khadi and Village Industry except once when he stated that 'Khadi of my conception' is that hand spun material which takes the place entirely, in India, of mill cloth...... and indirectly explained what is Khadi. If men and women will not take to hand spinning as a sacred duty, that is, the same person will not do carding, slivering and spinning, there is little hope for Khadi.

The question of market, method of production by machine tool or technology and credit etc. which have occupied a major significance in the post-independent period of rural industrialization, did not have IMAGERUNNERS a place in the initial stages, 8860450330 but came into the picture during the post independence period and Gandhi's stand on the issues have undergone change.

The question of a market for Khadi was not significant to Gandhi. In the sense, that Khadi was con- ceived with a much more ambitious object i.e. to make our villages starvation-proof. He believed that, "This is impossible unless the villages will wear Khadi themselves, sending only the surplus to the cities. The singular secret of Khadi lies in its salability in the place of its production and use to the manufacturers themselves". However, finding problems for a market for Khadi, Gandhi in 1946, accepted "Commercial Khadi" as a "go-cart". He said, "We ourselves are responsible for the creation of this problem, we did not know the science of Khadi, we do not know it fully even now. Therefore, like children, we stumble again and again and thereby learn to work. In order that we may not fall so as never to rise again we made use of a go-cart and are still using it". In so far as the village industries are concerned Gandhi believed that the question of demand does not arise as the expansion of village industries is related to demand which did exist in the villages. He said, "Given the demand, there is no doubt that most of our wants can be supplied from our villages".

Gandhi insisted on "Primitive methods" of production in the village industry and explained prelims.insightsonindia.com 39 © Insights Active Learning | All rights reserved - 133617. You may not reproduce, distribute or exploit the contents in any form without written permission by copyright owner. Copyright infringers may face civil and criminal liability Online Prelims Thematic TEST - 10 ( InsightsIAS Mock Test Series for UPSC Preliminary Exam 2020 )

that, "I suggest the return because there is no other way of giving employment to the millions of villagers who live in idleness". Mechanization he regarded as evil in view of more hands than required in work. However, in the later periods, Gandhi accepted the role of small equipment, machines, tools and technology, which should not replace labour but reduce the cost and drudgery of labour and increase efficiency of labour. Thus, Gandhian design of rural industrialization was developed in the passage of time.

Q Source: Based on CSP papers + https://www.mkgandhi.org/articles/G%20and%20the%2021st%20century.htm

51 Consider the following about Artificial gene synthesis. 1. It requires a template DNA base to be able to synthesize any other DNA. 2. Such gene synthesis can be used to combine chromosomes and genomes. 3. It employs the technique of DNA printing.

Which of the above is/are correct? A. 1 only B. 2 and 3 only C. 3 only D. 2 only

Your Answer : Correct Answer : B

Answer Justification :

Justification: Artificial gene synthesis, sometimes known as DNA printing is a method in synthetic biology that is used to create artificial genes in the laboratory. The method has been used to generate functional bacterial or yeast chromosomes containing approximately one million base pairs.

Unlike IMAGERUNNERS DNA synthesis in living 8860450330 cells, artificial gene synthesis does not require template DNA, allowing virtually any DNA sequence to be synthesized in the laboratory. It comprises two main steps, the first of which is solid-phase DNA synthesis, sometimes known as DNA printing.

This produces oligonucleotide fragments that are generally under 200 base pairs. The second step then involves connecting these oligonucleotide fragments using various DNA assembly methods. Because artificial gene synthesis does not require template DNA, it is theoretically possible to make a completely synthetic DNA molecules with no limits on the nucleotide sequence or size.

More recently, artificial gene synthesis methods have been developed that will allow the assembly of entire chromosomes and genomes. The first synthetic yeast chromosome was synthesised in 2014, and entire functional bacterial chromosomes have also been synthesised. In addition, artificial gene synthesis could in the future make use of novel nucleobase pairs (unnatural base pairs).

See https://www2.le.ac.uk/projects/vgec/schoolsandcolleges/topics/recombinanttechniques and https://e n.wikipedia.org/wiki/Artificial_gene_synthesis prelims.insightsonindia.com 40 © Insights Active Learning | All rights reserved - 133617. You may not reproduce, distribute or exploit the contents in any form without written permission by copyright owner. Copyright infringers may face civil and criminal liability Online Prelims Thematic TEST - 10 ( InsightsIAS Mock Test Series for UPSC Preliminary Exam 2020 )

Q Source: AR: UPSC CSP 2019

52 Which of these political views were held by Jayprakash Narayan? 1. The model of representative democracy in India had a tendency towards extreme devolution of power as against centralization. 2. A decentralized authority invariably fosters large impersonal bureaucracies and huge interest groups that reduce overall efficiency.

Which of the above is/are correct? A. 1 only B. 2 only C. Both 1 and 2 D. None

Your Answer : Correct Answer : D

Answer Justification :

Justification: He gave the concept of a partyless democracy, which has been explained well here http://vle.du.ac.in/file.php/617/Jayaprakash_Narayan_on_Democracy/Jayaprakash_Narayan_on_Dem ocracy.pdf

As per him, the spectrum of Indian political system remained stretched between the nation state on one end and the people on the other extreme.

The intermediary space seems to be completely devoid of any democratic mechanism.

In effect, the self-governing powers of the local bodies were severely compromised.

As a consequence of centralization of power in bureaucracy may soon lead to the autocracy of the bureaucrats, which is always a threat to autonomy and freedom IMAGERUNNERSwithin any representative 8860450330 democracy. Likewise, JP also firmly believed that the centralized authority invariably fosters vast impersonal bureaucracies and huge interest groups that make organic integration impossible. JP believed the remedy for this problem that the parliamentary democracy is rid with is actually to replace it with a communitarian democracy and decentralized political system. He wanted to avoid competitiveness because it is necessarily exploitative; and to achieve instead a cooperative and co-sharing integrated social order in which there would be a true harmonization of interests. Only decentralization of authority involved in the decision making process of all aspects of social development, may genuinely realize the idea of a communitarian democracy that JP believed in.

This communitarian idea of democracy that JP advocated according to him, would be a most participating democracy as it would have the capacity to integrate the entire system from top to bottom.

prelims.insightsonindia.com 41 © Insights Active Learning | All rights reserved - 133617. You may not reproduce, distribute or exploit the contents in any form without written permission by copyright owner. Copyright infringers may face civil and criminal liability Online Prelims Thematic TEST - 10 ( InsightsIAS Mock Test Series for UPSC Preliminary Exam 2020 )

At the village level this would translate into local self-government idea of Panchayati Raj and at the top it would become a completely partyless democracy.

Q Source: AR: UPSC CAPF 2017

53 Consider the following about Secondary Rainbows. 1. A secondary rainbow appears outside of a primary rainbow and develops when light entering a raindrop undergoes two internal reflections instead of just one. 2. A secondary rainbow is always brighter than a primary rainbow.

Select the correct answer using the codes below. A. 1 only B. 2 only C. Both 1 and 2 D. None

Your Answer : Correct Answer : A

Answer Justification :

Justification: A secondary rainbow appears outside of a primary rainbow and develops when light entering a raindrop undergoes two internal reflections instead of just one (as is the case with a primary rainbow).

The intensity of light is reduced even further by the second reflection, so secondary rainbows are not as bright as primary rainbows.

Alternatively: fewer light rays go through the four-step sequence than the three-step sequence.

IMAGERUNNERS 8860450330

The colour scheme of the secondary rainbow is opposite of the primary rainbow. Violet light from the higher drop enters the observer's eye, while red light from the same drop is incident elsewhere.

prelims.insightsonindia.com 42 © Insights Active Learning | All rights reserved - 133617. You may not reproduce, distribute or exploit the contents in any form without written permission by copyright owner. Copyright infringers may face civil and criminal liability Online Prelims Thematic TEST - 10 ( InsightsIAS Mock Test Series for UPSC Preliminary Exam 2020 )

Simultaneously, red light from the lower drop enters the observer's eye and violet light is not seen. This is why the colours of a secondary rainbow change from violet on the top to red on the bottom.

Q Source: AR: UPSC CAPF 2017

54 The hills contain seven peaks namely, Anjanadri, Garudadri, Narayanadri, Neeladri, Seshadri, Venkatadri and Vrishabhadri. A decade before, it was declared as a Biosphere Reserve:

A. Agasthyamala B. Seshachalam C. Velliangiri D. Annamalai

Your Answer : Correct Answer : B

Answer Justification :

Justification: IMAGERUNNERS The hills contain 8860450330 seven peaks namely, Anjanadri, Garudadri, Narayanadri, Neeladri, Seshadri, Venkatadri and Vrishabhadri, the highest at about 600 m above sea level.

The seven peaks are said to represent the seven hoods of Lord Adisesha, the king of serpents in Hindu mythology. The famous Natural Arch, Tirumala Hills is also a part of Seshachalam Hills, which dates back to the period in between Middle and Upper Proterozoic Eon. In 2010 it was designated as a Biosphere Reserve.

Q Source: Based on UPSC CSP papers 2017

55 Consider the following about the English Education Act, 1835. 1. It intended to give effect to the decision of Lord William Bentick, the then Governor General of British India. 2. It sought the promotion of oriental studies in Indian subjects. 3. It provided that the British Government ought to promote rational thinking among the natives of prelims.insightsonindia.com 43 © Insights Active Learning | All rights reserved - 133617. You may not reproduce, distribute or exploit the contents in any form without written permission by copyright owner. Copyright infringers may face civil and criminal liability Online Prelims Thematic TEST - 10 ( InsightsIAS Mock Test Series for UPSC Preliminary Exam 2020 )

India.

Which of the above is/are correct? A. 1 only B. 2 and 3 only C. 1 and 3 only D. 1, 2 and 3

Your Answer : Correct Answer : C

Answer Justification :

Justification: It was a legislative Act of the Council of India in 1835 giving effect to a decision in 1835 by Lord William Bentinck, the then Governor- General of British India, to reallocate funds the East India Company was required by the British Parliament to spend on education and literature in India.

It was based on the belief that the British Government ought to be promoting European literature and science among the natives of India; and that all the funds appropriated for the purpose of education would be best employed on English education alone.

Formerly, they had supported traditional Muslim and Hindu education and the publication of literature in the native learned tongues (Sanskrit and Persian); henceforward they were to support establishments teaching a Western curriculum with English as the language of instruction.

Together with other measures promoting English as the language of administration and of the higher law courts (replacing Persian), this led eventually to English becoming one of the languages of India, rather than simply the native tongue of its foreign rulers.

Q Source: AR: UPSC CAPF 2017

56 Which IMAGERUNNERS of the following is NOT categorized 8860450330 as a ‘Sugar’ despite being a carbohydrate?

A. Galactose B. Cellulose C. Lactose D. Fructose

Your Answer : Correct Answer : B

Answer Justification :

Justification & Learning: Carbohydrates are polyhydroxy aldehydes, ketones, alcohols, acids, their simple derivatives and their polymers.

They may be classified according to their degree of polymerization and may be divided initially into prelims.insightsonindia.com 44 © Insights Active Learning | All rights reserved - 133617. You may not reproduce, distribute or exploit the contents in any form without written permission by copyright owner. Copyright infringers may face civil and criminal liability Online Prelims Thematic TEST - 10 ( InsightsIAS Mock Test Series for UPSC Preliminary Exam 2020 )

three principal groups, namely sugars, oligosaccharides and polysaccharides.

Even though Cellulose is a carbohydrate, it is not a Sugar.

Q Source: AR: Past year UPSC papers: CDS 2014

57 Consider the following statements. 1. India’s total food grain production in 2019-20 is greater than that of the average production in previous five years’ (2014-15 to 2018-19). 2. In terms of volume of production, the most produced crop was rice.

Which of the above is/are correct? A. 1 only B. 2 only C. Both 1 and 2 D. None

Your Answer : Correct Answer : A

Answer IMAGERUNNERS Justification : 8860450330

Learning: India has harvest record food grains, oil-seeds, pulses and cotton during 2019-2020.

The total food grain production in the country is estimated at record 295.67 million tonnes which is higher by 10.46 MT than 285.21 MT achieved during 2018-19, according to the third advance estimates for the crop year released by the Agriculture Ministry on Friday.

The production during 2019-20 is higher by 25.89 MT than the average production in previous five years’ (2014-15 to 2018-19), it said.

The cumulative rainfall in the country during the 2019 monsoon season (June to September) was 10% higher than Long Period Average (LPA).

Accordingly, the production of most of the crops for the agricultural year 2019-20 has been estimated higher than their normal production, as per the Agriculture Ministry.

prelims.insightsonindia.com 45 © Insights Active Learning | All rights reserved - 133617. You may not reproduce, distribute or exploit the contents in any form without written permission by copyright owner. Copyright infringers may face civil and criminal liability Online Prelims Thematic TEST - 10 ( InsightsIAS Mock Test Series for UPSC Preliminary Exam 2020 )

Sugarcane during 2019-20 is estimated at 358.14 MT while cotton at 36.05 million bales (of 170 kg each) is higher by 8.01 million bales than 28.04 million bales during 2018-19.

Q Source: Based on UPSC CSP papers

58 Capital gains tax in India will be applicable on an increase in market price of the following if held by an individual: 1. Antique collections 2. Gold Jewellery 3. A modern painting 4. Shares of companies

Select the correct answer using the codes below. A. 1 and 2 only B. 4 only C. 2 and 4 only D. 1, 2, 3 and 4

Your Answer : Correct Answer : D

Answer Justification :

Justification: Profits or gains arising from transfer of a capital asset are called “Capital Gains” and are charged to tax under the head “Capital Gains”.

Capital asset is defined to include: Any kind of property held by an assessee, whether or not connected with business or profession of the assesse, stocks, movable property, jewellery; archaeological collections; drawings; paintings; sculptures; or any work of art.

For e.g. if you purchased a residential house in 2015 for Rs. 10,00,000 and sold it in 2016 for Rs. 20,00,000, a capital asset gain of Rs. 10,00,000 arising on account of sale of residential house will be chargedIMAGERUNNERS to tax under the head 8860450330 “Capital Gains”.

See http://www.incometaxindia.gov.in/Tutorials/15-%20LTCG.pdf

Q Source: Based on CSP 2011-13

59 Sustainable Development Knowledge Platform (SDKP) is an initiative of

A. WWF (World Wildlife Fund) B. International Union for Conservation of Nature (IUCN) C. Conservation International D. United Nations

Your Answer : Correct Answer : D prelims.insightsonindia.com 46 © Insights Active Learning | All rights reserved - 133617. You may not reproduce, distribute or exploit the contents in any form without written permission by copyright owner. Copyright infringers may face civil and criminal liability Online Prelims Thematic TEST - 10 ( InsightsIAS Mock Test Series for UPSC Preliminary Exam 2020 )

Answer Justification :

Learning: The UN Department of Economic and Social Affairs (UNDESA) launched the UN web- based knowledge platform for sustainable development (Sustainable Development Knowledge Platform SDKP).

The site describes different sustainable development activities within the UN system, and it provides resources and links to nearly 6,000 related documents.

The platform presents analysis on the key topics including in the Rio+20 outcome document and contains additional links related to Agenda 21 and the Commission for Sustainable Development. See https://sustainabledevelopment.un.org/

Q Source: Based on UPSC CSP papers

60 With reference to Chaitanya, consider the following statements. 1. He was a Bhakti Saint in Medieval Bengal. 2. He was against renunciation and asceticism. 3. He believed that love and devotion can help one feel the presence of God.

Select the correct answer using the codes below. A. 1 and 2 only B. 3 only C. 1 and 3 only D. 2 and 3 only

Your Answer : Correct Answer : C

Answer Justification :

Justification: He was a well-known saint and reformer of Bengal who popularised the Krishna cult. He IMAGERUNNERSrenounced the material world, 8860450330 became an ascetic and wandered all over the country preaching his ideas.

He proclaimed the universal brotherhood of man and condemned all distinction based on religion and caste. He emphasised love and peace and showed great sympathy to the sufferings of other people, especially that of the poor and the weak.

He believed that through love and devotion, song and dance, a devotee can feel the presence of God. He accepted disciples from all classes and castes and his teachings are widely followed in Bengal even today.

Q Source: Based on CSP papers

61 Amongst the following, identify the areas with inadequate rainfall (<50 cm annually).

A. Southern parts of Gujarat prelims.insightsonindia.com 47 © Insights Active Learning | All rights reserved - 133617. You may not reproduce, distribute or exploit the contents in any form without written permission by copyright owner. Copyright infringers may face civil and criminal liability Online Prelims Thematic TEST - 10 ( InsightsIAS Mock Test Series for UPSC Preliminary Exam 2020 )

B. Western Rajasthan C. North-eastern Peninsula - Orissa and Jharkhand D. Cachar valley (Assam) and most of the regions of Manipur

Your Answer : Correct Answer : B

Answer Justification :

Justification & Learning: Areas of High Rainfall : The highest rainfall occurs along the west coast, on the Western Ghats, as well as in the sub-Himalayan areas is the northeast and the hills of Meghalaya. Here the rainfall exceeds 200 cm. In some parts of Khasi and Jaintia hills, the rainfall exceeds 1,000 cm. In the Brahmaputra valley and the adjoining hills, the rainfall is less then 200 cm.

Areas of Medium Rainfall: Rainfall between 100-200 cm is received in the southern parts of Gujarat, east Tamil Nadu, northeastern Peninsula covering Orissa, Jharkhand, Bihar, eastern Madhya Pradesh, northern Ganga plain along the sub-Himalayas and the Cachar Valley and Manipur.

Areas of Low Rainfall: Western Uttar Pradesh, Delhi, Haryana, Punjab, Jammu and Kashmir, eastern Rajasthan, Gujarat and Deccan Plateau receive rainfall between 50-100 cm.

Areas of Inadequate Rainfall: Parts of the Peninsula, especially in Andhra Pradesh, Karnataka and Maharashtra, Ladakh and most of western Rajasthan receive rainfall below 50 cm.

Q Source: Based on CSP papers

62 MoSAic International Observatory studies

A. Amazonian Rainforests B. Deep space C. Arctic Region D.IMAGERUNNERSUnderground limestone 8860450330caves

Your Answer : Correct Answer : C

Answer Justification :

Background and Learning: The Arctic is a key area of global climate change, with warming rates exceeding twice the global average.

Many processes in the Arctic climate system are poorly represented in climate models because they are not sufficiently understood. Understanding of Arctic climate processes is limited by a lack of year round observations in the central Arctic. The Multidisciplinary drifting Observatory for the study of Arctic climate (MOSAiC) will be the first year-round expedition into the central Arctic exploring the Arctic climate system. prelims.insightsonindia.com 48 © Insights Active Learning | All rights reserved - 133617. You may not reproduce, distribute or exploit the contents in any form without written permission by copyright owner. Copyright infringers may face civil and criminal liability Online Prelims Thematic TEST - 10 ( InsightsIAS Mock Test Series for UPSC Preliminary Exam 2020 )

The project with a total budget exceeding 60 Million € has been designed by an international consortium of leading polar research institutions under the umbrella of the International Arctic Science Committee (IASC). The results of MOSAiC will contribute to enhance understanding of the regional and global consequences of Arctic climate change and sea-ice loss and improve weather and climate predictions. As such it will support safer maritime and offshore operations, contribute to an improved scientific basis for future fishery and traffic along northern sea routes, increase coastal- community resilience, and support science-informed decision-making and policy development.

See http://www.mosaicobservatory.org/

Q Source: Based on CSP papers

63 Match the following rivers with their state of origin: 1. Tapti: Madhya Pradesh 2. Godavari: Andhra Pradesh 3. Cauvery: Kerala 4. Mahanadi : Chhattisgarh

Select the correct answer using the codes below. A. 1 and 4 only B. 2 and 3 only C. 1, 2 and 3 only D. 1, 3 and 4 only

Your Answer : Correct Answer : A

Answer Justification :

Justification: IMAGERUNNERS Statement 1: The 8860450330 Tapti River between the Godavari and Narmada rivers starts from Betul, Madhya Pradesh, and then flows westwards before draining through the Gulf of Khambhat into the Arabian Sea.

Statement 2: The Godavari is India's second longest river after the Ganga. Its source is in Triambakeshwar, Maharashtra.

Statement 3: Originating in the foothills of Western Ghats at Talakaveri, Kodagu in Karnataka Cauvery flows generally south and east through Karnataka and Tamil Nadu emptying into the Bay of Bengal.

Statement 4: Mahanadi is a combination of many mountain streams and thus its precise source is impossible to pinpoint. However its farthest headwaters lie 6 kilometres from Pharsiya village in Dhamtari district of Chhattisgarh.

Q Source: Based on UPSC CSP papers

prelims.insightsonindia.com 49 © Insights Active Learning | All rights reserved - 133617. You may not reproduce, distribute or exploit the contents in any form without written permission by copyright owner. Copyright infringers may face civil and criminal liability Online Prelims Thematic TEST - 10 ( InsightsIAS Mock Test Series for UPSC Preliminary Exam 2020 )

64 Consider the following statements. 1. The Ramayana mentions chariots covered with leather. 2. The Rig Vedic charioteers used varma (coats of mail) and sipra/sironastra (helmets). 3. The Rig Veda mentions horse-drawn chariots with spoked wheels.

Select the correct answer using the codes below. A. 1 and 2 only B. 2 and 3 only C. 3 only D. 1, 2 and 3

Your Answer : Correct Answer : D

Answer Justification :

Learning: The Ramayana mentions chariots covered with leather. The Rig Vedic charioteers used varma (coats of mail) and sipra/sironastra (helmets).

Equipped with asi (swords), hanas (arrows) and ilhianus (bows), the Kshatriyas on the chariots went to combat.

The Rig Veda mentions horse-drawn chariots with spoked wheels.

As for the use of horse in later Indus valley civilizations, some scholars claim that the horse was widely domesticated and used in India in the area covered by the Indus-Sarasvati (or Harappan) Civilisation, but this evidence remains unsubstantiated.

This question is adopted from a question on the use of horses in Indus valley civilization.

Q Source: Based on past CSP paper: 2017

65 Which IMAGERUNNERS of the following are under 8860450330 the roles/ functions of the National Afforestation and Eco- Development Board (NAEB)? 1. Evolve mechanisms for ecological restoration of degraded forest areas 2. Rehabilitating and resettling communities displaced by deforestation for developmental works 3. Sponsor research and extension of research findings to disseminate new technologies for the regeneration and development of degraded forest

Select the correct answer using the codes below. A. 1 and 2 only B. 3 only C. 1 and 3 only D. 2 and 3 only

Your Answer : Correct Answer : C

prelims.insightsonindia.com 50 © Insights Active Learning | All rights reserved - 133617. You may not reproduce, distribute or exploit the contents in any form without written permission by copyright owner. Copyright infringers may face civil and criminal liability Online Prelims Thematic TEST - 10 ( InsightsIAS Mock Test Series for UPSC Preliminary Exam 2020 )

Answer Justification :

Justification: Set up in 1992, the detailed role and functions of the NAEB are to:

Evolve mechanisms for ecological restoration of degraded forest areas and adjoining lands through systematic planning and implementation, in a cost effective manner; Restore through natural regeneration or appropriate intervention the forest cover in the country for ecological security and to meet the fuelwood, fodder and other needs of the rural communities; Restore fuelwood, fodder, timber and other forest produce on the degraded forest and adjoining lands in order to meet the demands for these items; Sponsor research and extension of research findings to disseminate new and proper technologies for the regeneration and development of degraded forest areas and adjoining lands; Create general awareness and help foster people's movement for promoting afforestation and eco-development with the assistance of voluntary agencies, non-government organisations, Panchayati Raj institutions and others and promote participatory and sustainable management of degraded forest areas and adjoining lands; Coordinate and monitor the Action Plans for afforestation, tree planting, ecological restoration and eco-development; and Undertake all other measures necessary for promoting afforestation, tree planting, ecological restoration and eco-development activities in the country.

Q Source: AR: UPSC past year papers

66 MARPOL Convention mainly deals internationally with

A. Pollution from Ships B. Civil Aviation Protocols C. Mining regulations D. Sharing of water resources IMAGERUNNERS 8860450330 Your Answer : Correct Answer : A

Answer Justification :

Learning: It is also known as International Convention for the Prevention of Pollution from Ships (MARPOL Convention).

It is the main international convention that deals with this subject. It was adopted in 1973 and entered into force in 1983.

Its biodiversity relevance stems from the highly detrimental effect that pollution can have on marine organisms, therefore by preventing the pollution of the sea the Convention contributes towards the maintenance of marine biodiversity.

Q Source: AR: UPSC past year papers prelims.insightsonindia.com 51 © Insights Active Learning | All rights reserved - 133617. You may not reproduce, distribute or exploit the contents in any form without written permission by copyright owner. Copyright infringers may face civil and criminal liability Online Prelims Thematic TEST - 10 ( InsightsIAS Mock Test Series for UPSC Preliminary Exam 2020 )

67 Consider the following matches of Buddhist Monasteries with their locations. 1. : Spiti 2. Kardang Monastery : Lahaul 3. Rumtek Monastery : Dehradun

Select the correct answer using the codes below. A. 1 and 2 only B. 3 only C. 1 only D. 2 and 3 only

Your Answer : Correct Answer : A

Answer Justification :

Learning: Statement 1: It is located in the Tabo village of , . It was founded in 996 CE in the Tibetan year of the Fire Ape.

Tabo is noted for being the oldest continuously operating Buddhist enclave in both India and the Himalayas. A large number of frescoes displayed on its walls depict tales from the Buddhist pantheon.

Statement 2: Kardang Monastery or Gompa is a famous monastery, and is the most important monastery the Lahaul valley, India. The monastery is a huge white building bedecked with prayer flags.

Statement 3: The monastery is currently the largest in Sikkim. It is home to the community of monks and where they perform the rituals and practices of the Karma lineage. A golden stupa IMAGERUNNERS contains the relics of the 8860450330 16th . Opposite that building is a college, Karma Shri Nalanda Institute for Higher Buddhist Studies.

Q Source: Based on UPSC past year papers

68 Consider the following about Aruna Asaf Ali. 1. She hoisted the Indian National Congress flag at the Gowalia Tank maidan in Bombay during the Quit India Movement, 1942. 2. She was a member of the Congress Socialist Party. 3. She participated in public processions during the Salt Satyagraha.

Which of the above is/are correct? A. 1 only B. 2 and 3 only C. 3 only D. 1, 2 and 3 prelims.insightsonindia.com 52 © Insights Active Learning | All rights reserved - 133617. You may not reproduce, distribute or exploit the contents in any form without written permission by copyright owner. Copyright infringers may face civil and criminal liability Online Prelims Thematic TEST - 10 ( InsightsIAS Mock Test Series for UPSC Preliminary Exam 2020 )

Your Answer : Correct Answer : D

Answer Justification :

Learning: She became an active member of Congress Party after marrying Asaf Ali and participated in public processions during the Salt Satyagraha.

In 1954, she helped form the National Federation of Indian Women, the women's wing of CPI but left the party in 1956 following Nikita Khrushchev's disowning of Stalin.

She received India's highest civilian award, the Bharat Ratna, posthumously in 1997. She was a member of the Congress Socialist Party

Q Source: Based on CSP 2015-17

69 Who among the following was/were associated with the Anushilan Samiti formed in 1906? 1. Aurobindo Ghosh 2. Rash Behari Bose 3. Jatindranath Mukherjee

Which of the above is/are correct? A. 1 only B. 2 and 3 only C. 3 only D. 1, 2 and 3

Your Answer : Correct Answer : D

Answer Justification :

Learning: IMAGERUNNERS It was a Bengali Indian 8860450330 organisation and arose from conglomerations of local youth groups and gyms in Bengal.

It had two prominent if somewhat independent arms in East and West Bengal identified as Dhaka Anushilan Samiti and the Jugantar group respectively.

Between its foundations to its gradual dissolution through 1930s, the Samiti collaborated with other revolutionary organisations in India and abroad.

Led by notable revolutionaries of the likes of Aurobindo Ghosh, Rash Behari Bose and Jatindranath Mukherjee, the Samiti was involved in a number of noted incidences of revolutionary terrorism against British interests and administration in India. These included the early attempts to assassinate Raj officials, the 1912 attempt on the life of Viceroy of India, as well as the Sedetious conspiracy during World War I.

Q Source: AR: UPSC past year papers prelims.insightsonindia.com 53 © Insights Active Learning | All rights reserved - 133617. You may not reproduce, distribute or exploit the contents in any form without written permission by copyright owner. Copyright infringers may face civil and criminal liability Online Prelims Thematic TEST - 10 ( InsightsIAS Mock Test Series for UPSC Preliminary Exam 2020 )

70 Consider the following about Bioremediation. 1. Bacteria, fungi and protists all can be used for bioremediation. 2. Bioremediation is not adequate for removing those toxic heavy metals since that could be toxic to the microbes.

Which of the above is/are correct? A. 1 only B. 2 only C. Both 1 and 2 D. None

Your Answer : Correct Answer : C

Answer Justification :

Learning: In a non-polluted environment, bacteria, fungi, protists, and other microorganisms are constantly at work breaking down organic matter, which we call as bioremediation.

If an organic pollutant such as oil contaminated this environment, some of the microorganisms would die, while others capable of eating the organic pollution would survive.

Bioremediation works by providing these pollution-eating organisms with fertilizer, oxygen, and other conditions that encourage their rapid growth.

However, bioremediation may not provide a feasible strategy at sites with high concentrations of chemicals that are toxic to most microorganisms. These chemicals include some toxic heavy metals such as cadmium or lead, and salts such as sodium chloride.

http://ei.cornell.edu/biodeg/bioremed/

Q Source: Based on UPSC CSP papers 2017 IMAGERUNNERS 8860450330 71 Food Safety Display Boards (FSDBs) are colour coded for different kind of food businesses for ease of recognition by the consumers. The colour designations as per the kind of business is: 1. Restaurant: Yellow 2. Fruit & Vegetable Retail: Green 3. Street Food: Purple 4. Retail Store: Grey

Select the correct answer using the codes below. A. 1 and 2 only B. 2 and 4 only C. 2, 3 and 4 only D. 1 and 3 only

Your Answer : Correct Answer : C prelims.insightsonindia.com 54 © Insights Active Learning | All rights reserved - 133617. You may not reproduce, distribute or exploit the contents in any form without written permission by copyright owner. Copyright infringers may face civil and criminal liability Online Prelims Thematic TEST - 10 ( InsightsIAS Mock Test Series for UPSC Preliminary Exam 2020 )

Answer Justification :

Justification: As per the FSS Regulation, there is a mandatory requirement of displaying FSSAI License/Registration Number at food premises. Usually, the FSSAI license number is not visible to the consumers.

Thus, to change the overall consumer perceptibility and to strengthen food safety, FSSAI has introduced Food Safety Display Boards (FSDBs) for various food businesses.

Henceforth, in addition to the existing mandatory requirement of displaying FSSAI License/ Registration Number, it will also be mandatory (in a phased manner) for FBOs to display these Food Safety Display Boards at FBO premise.

The FSDBs are colour coded for different kind of food businesses for ease of recognition by the consumers. The colour designation as per kind of business is as follows :-

Restaurant (Purple) Fruit & Vegetable Retail (Green) Meat Retail (Red) Milk Retail (Blue) Street Food (Purple) Retail Store (Grey)

See http://www.fssai.gov.in/home/safe-food-practices/food-safety-display-boards.html

Q Source: Based on UPSC CSP papers 2016

72 Geo-sequestration is a method that involves injecting carbon dioxide, generally in supercritical form, directly into underground geological formations. Which of these sites can be used for the same? 1. Oil fields 2. Saline-filled basalt formations 3. Unmineable coal seams IMAGERUNNERS 8860450330 Which of the above is/are correct? A. 1, 2 and 3 B. 2 only C. 1 and 2 only D. 1 and 3 only

Your Answer : Correct Answer : A

Answer Justification :

Justification: Of the enhanced oil recovery methods currently being considered for application to many of the nation’s older oil fields, carbon dioxide flooding may offer the largest potential for additional oil recovery

CO2 is sometimes injected into declining oil fields to increase oil recovery. prelims.insightsonindia.com 55 © Insights Active Learning | All rights reserved - 133617. You may not reproduce, distribute or exploit the contents in any form without written permission by copyright owner. Copyright infringers may face civil and criminal liability Online Prelims Thematic TEST - 10 ( InsightsIAS Mock Test Series for UPSC Preliminary Exam 2020 )

Other potential sites are gas fields, saline formations, unmineable coal seams, and saline-filled basalt formations can act as storage sites. After sequestration, various physical (e.g., highly impermeable caprock) and geochemical trapping mechanisms prevent the CO2 from escaping to the surface.

Q Source: Based on UPSC CSP papers 2017

73 Consider the following statements. #95818 1. The term "transcriptome" can also be used to describe the array of mRNA transcripts produced in a particular cell or tissue type. 2. An organism's transcriptome remains static or unchanged throughout its life. 3. Transcriptomes can be used to infer phylogenetic relationships among individuals.

Which of the above is/are correct? A. 1 only B. 2 and 3 only C. 1 and 3 only D. 1, 2 and 3

Your Answer : Correct Answer : C

Answer Justification :

Justification: Statement 1: A transcriptome is the full range of messenger RNA, or mRNA, molecules expressed by an organism.

Statement 2: In contrast with the genome, which is characterized by its stability, the transcriptome actively changes.

In fact, an organism's transcriptome varies depending on many factors, including stage of development IMAGERUNNERS and environmental 8860450330 conditions.

Statement 3: Applications: The transcriptomes of stem cells and cancer cells are of particular interest to researchers who seek to understand the processes of cellular differentiation and carcinogenesis.

Analysis of the transcriptomes of human oocytes and embryos is used to understand the molecular mechanisms and signaling pathways controlling early embryonic development, and could theoretically be a powerful tool in making proper embryo selection in in vitro fertilisation. Transcriptomics is an emerging and continually growing field in biomarker discovery for use in assessing the safety of drugs or chemical risk assessment. Transcriptomes may also be used to infer phylogenetic relationships among individuals.

See http://www.nature.com/scitable/definition/transcriptome-296 prelims.insightsonindia.com 56 © Insights Active Learning | All rights reserved - 133617. You may not reproduce, distribute or exploit the contents in any form without written permission by copyright owner. Copyright infringers may face civil and criminal liability Online Prelims Thematic TEST - 10 ( InsightsIAS Mock Test Series for UPSC Preliminary Exam 2020 )

Q Source: Based on UPSC CSP papers 2016

74 Consider the following matches of these places with the religious structures: Places: Religious structure 1. Pavurallakonda: A. Chudamani Vihara 2. Bheemunipatnam: B. Narasimha temple 3. Murudeshwar: C. One of the tallest Shiva statues

Select the correct matches using the codes below. A. 1A, 2B, 3C B. 1B, 2A, 3C C. 1C, 2B, 3A D. 1B, 2C, 3A

Your Answer : Correct Answer : B

Answer Justification :

Justification: Statement 1: There is a Hindu temple dedicated to Lord Narasimha on the eastern side of Pavurallakonda facing the sea. As per the historical evidences the temple is constructed around 14th century by Mindi Kings.

Statement 2:Bheemunipatnam is a neighborhood of Visakhapatnam in the Indian state of Andhra Pradesh.

The town came to prominence during the period of Medieval Cholas (9th–12th century CE) and served as their important port for commerce and east-bound naval expeditions. The ChudamaniVihara in Nagapattinam constructed by the Sri Lankan king with the help of IMAGERUNNERSChola kingdom is an important 8860450330 Buddhist structure of the times. Nagapattinam was settled by the Portuguese and, later, the Dutch under whom it served as the capital of Dutch Coromandel from 1660 to 1781 Buddhist remains were found during the excavations here along with Relic caskets. Both the Hinayana and Mahayana were propagated in this land and also to the far east lands from the port of River Gosthani.

Statement 3: Murdeshwar is a town in Bhatkal Taluk of Uttara Kannada district in the state of Karnataka, India. The town is located 13 km from the taluk headquarters of Bhatkal. Murdeshwar is famous for the world's second tallest Shiva statue, the town lies on the coast of the Arabian Sea and is also famous for the Murdeshwar Temple.

Q Source: Based on UPSC CSP papers 2017

75 In particle physics (Standard Model), an elementary particle or fundamental particle is a subatomic particle with no substructure – what does this mean? prelims.insightsonindia.com 57 © Insights Active Learning | All rights reserved - 133617. You may not reproduce, distribute or exploit the contents in any form without written permission by copyright owner. Copyright infringers may face civil and criminal liability Online Prelims Thematic TEST - 10 ( InsightsIAS Mock Test Series for UPSC Preliminary Exam 2020 )

A. the particle has no spin B. the particle has no energy C. the particle is not composed of any other particles D. the particle is massless

Your Answer : Correct Answer : C

Answer Justification :

Justification: In particle physics, an elementary particle or fundamental particle is a subatomic particle with no substructure, i.e. it is not composed of other particles.

Particles currently thought to be elementary include the fundamental fermions (quarks, leptons, antiquarks, and antileptons), which generally are "matter particles" and "antimatter particles", as well as the fundamental bosons (gauge bosons and the Higgs boson), which generally are "force particles" that mediate interactions among fermions. A particle containing two or more elementary particles is called a composite particle.

As per the Standard Model of particle physics:

All matter around us is made of elementary particles, the building blocks of matter. These particles occur in two basic types called quarks and leptons.

There are four fundamental forces at work in the universe: the strong force, the weak force, the electromagnetic force, and the gravitational force. They work over different ranges and have different strengths. Gravity is the weakest but it has an infinite range. The electromagnetic force also has infinite range but it is many times stronger than gravity

Three of the fundamental forces result from the exchange of force-carrier particles, which belong to a broader group called “bosons”.

Q Source: Based on UPSC CSP papers 2017 IMAGERUNNERS 8860450330

76 The Centre for the Fourth Industrial Revolution is an initiative of the

A. Organisation for Economic Co-operation and Development (OECD) B. International Labour Organization (ILO) C. North American Free Trade Area (NAFTA) D. World Economic Forum (WEP)

Your Answer : Correct Answer : D

Answer Justification :

Learning: It is a hub for global, multi-stakeholder cooperation to develop policy frameworks and advance collaborations that accelerate the benefits of science and technology. prelims.insightsonindia.com 58 © Insights Active Learning | All rights reserved - 133617. You may not reproduce, distribute or exploit the contents in any form without written permission by copyright owner. Copyright infringers may face civil and criminal liability Online Prelims Thematic TEST - 10 ( InsightsIAS Mock Test Series for UPSC Preliminary Exam 2020 )

Policy governing emerging technologies is currently piecemeal: Some areas are regulated heavily, others hardly at all. Often, mechanisms for policy-makers to interact with people at the cutting edge of research simply do not exist.

There is a need for a global and trusted space where leading technology companies, dynamic start- ups, policy-makers etc can collaborate to develop the agile policy norms and partnerships needed to stimulate the enormous potential of science and technology.

Q Source: Based on UPSC CSP papers 2017

77 Consider the following matches of awards for the categories they are known for: 1. Man Booker Prize: Non-fiction 2. Pulitzer Prize: Fiction 3. Specsavers National Book Awards: Scientific Journals

Select the correct matches using the codes below. A. 1 and 2 only B. 2 only C. 2 and 3 only D. 1 and 3 only

Your Answer : Correct Answer : B

Answer Justification :

Justification: Statement 1: The Man Booker Prize for Fiction is awarded to writers of full-length novels from the Commonwealth of Nations, Ireland, or Zimbabwe. 2016's winner was Paul Beatty and past winners have included Hilary Mantel for Bring Up the Bodies, Margaret Atwood for The Blind Assassin, Kazuo Ishiguro for The Remains of the Day and William Golding for Rites of Passage.

Statement IMAGERUNNERS 2: This US award is8860450330 given to those in newspaper or online journalism, literature, or music. There are 21 categories including the Pulitzer Prize for Editorial Writing, the Pulitzer Prize for Music and the Pulitzer Prize for Fiction.

Statement 3: The Specsavers National Book Awards includes many categories, such as the Book of the Year, Outstanding Achievement, UK Author of the Year, Children's Book of the Year and Thriller & Crime Novel of the Year.

Q Source: AR: UPSC CAPF 2017

78 Which of the following species have been provided protection under Schedule I of the Wildlife (Protection) Act, 1972? 1. Gangetic Dolphin 2. India Gazelle 3. Lion-tailed Macaque 4. Pygmy Hog prelims.insightsonindia.com 59 © Insights Active Learning | All rights reserved - 133617. You may not reproduce, distribute or exploit the contents in any form without written permission by copyright owner. Copyright infringers may face civil and criminal liability Online Prelims Thematic TEST - 10 ( InsightsIAS Mock Test Series for UPSC Preliminary Exam 2020 )

Select the correct answer using the code given below: A. 1 only B. 2 and 3 only C. 1, 3 and 4 only D. 1, 2, 3 and 4

Your Answer : Correct Answer : D

Answer Justification :

Justification: All of these are protected scheduled animals under the Wildlife Act under Schedule I. Strictest protection is provided under the Schedule I with protection levels declining in successive schedules.

The whole list can be seen here: http://envfor.nic.in/legis/wildlife/wildlife2s1.pdf

Q Source: Based on UPSC CSP papers 2017

79 Which of the following is a tributary of Yamuna river?

A. Ghaghara B. Gandak C. Mandakini D. Hindon

Your Answer : Correct Answer : D

Answer Justification :

Learning: IMAGERUNNERS Son, Ghaghara, Gomti, 8860450330 Yamuna and Gandak are some of the most important tributaries of Ganga.

The important tributaries of the Yamuna River are Tons, Chambal, Hindon, Betwa and Ken. Other small tributaries of the Yamuna River include the Giri, Sind, Uttangan, Sengar and the Rind.

Brahmani is a tributary of Mahanadi. Others are Ib, Jonk, Tel, HAsdep, Kuakhai etc.

Q Source: Based on UPSC CSP papers

80 A UPC, short for Universal Product Code, is a type of code printed on Retail product packaging. Consider the following about it. 1. It consists of a QR code and a unique 8 digit number below it. 2. It is helpful in tracking inventory within a store or warehouse. 3. It can be used to obtain some information about the product that carries the UPC.

prelims.insightsonindia.com 60 © Insights Active Learning | All rights reserved - 133617. You may not reproduce, distribute or exploit the contents in any form without written permission by copyright owner. Copyright infringers may face civil and criminal liability Online Prelims Thematic TEST - 10 ( InsightsIAS Mock Test Series for UPSC Preliminary Exam 2020 )

Select the correct answer using the codes below. A. 1 and 2 only B. 2 and 3 only C. 1 and 3 only D. 1, 2 and 3

Your Answer : Correct Answer : B

Answer Justification :

Justification: It aids in identifying a particular item. It consists of two parts – the machine-readable barcode, which is a series of unique black bars, and the unique 12-digit number beneath it.

The purpose of UPCs is to make it easy to identify product features, such as the brand name, item, size, and color, when an item is scanned at checkout. In fact, that’s why they were created in the first place – to speed up the checkout process at grocery stores. UPCs are also helpful in tracking inventory within a store or warehouse.

Q Source: Based on UPSC CSP papers

81 The Euro-Atlantic Partnership Council (EAPC) is most closely associated with the

A. North-Atlantic Treaty Organization (NATO) B. Shanghai Cooperation Organization (SCO) C. Organization of the Petroleum Exporting Countries (OPEC) D. International Monetary Fund (IMF)

Your Answer : Correct Answer : A

Answer IMAGERUNNERS Justification : 8860450330

Learning: It is a post-Cold War NATO institution, is a multilateral forum created to improve relations between NATO and non-NATO countries in Europe and those parts of Asia on the European periphery.

States meet to cooperate and go to the range of political and security issues.

It was formed in 1997 in the ministers meeting of Sintra, Portugal, as the successor to the North Atlantic Cooperation Council (NACC), which was created in 1991. It works alongside the Partnership for Peace (PfP), created in 1994.

Q Source: Based on UPSC CSP papers

82 Consider the following statements. 1. India is the only country in the world to produce all the four known varieties of silk including prelims.insightsonindia.com 61 © Insights Active Learning | All rights reserved - 133617. You may not reproduce, distribute or exploit the contents in any form without written permission by copyright owner. Copyright infringers may face civil and criminal liability Online Prelims Thematic TEST - 10 ( InsightsIAS Mock Test Series for UPSC Preliminary Exam 2020 )

Mulberry, Eri, Tasar and Muga. 2. Mulberry is the largest used raw material in sericulture industry accounting for almost three-fourth of the entire silk production in India.

Which of the above is/are correct? A. 1 only B. 2 only C. Both 1 and 2 D. None

Your Answer : Correct Answer : C

Answer Justification :

Learning: Today India is the second largest silk producer of raw silk and also has the distinction of being the world’s largest consumer of pure silk.

India is the only country in the world to produce all the four known varieties of silk including Mulberry, Eri, Tasar and Muga. Mulberry is the largest practiced sericulture industry accounting for almost 76 per cent of the entire silk production The country is known the world over for the exquisite brocade fabrics of Banaras, silks of Karnataka, tie-and–dye and Patola of Gujarat and Rajasthan, ikats from Orissa, fine Bandhej and temple silks of Kancheepuram and Tanjore, etc., are only a few of the myriad range of silk weaves, textures and patterns available in India.

Q Source: Based on UPSC CSP papers

83 Which of these protected areas is also famously known as ‘Top Slip’ due to an old practice of sliding timber logs down the hill from here:

A.IMAGERUNNERSBandipur National Park 8860450330 B. Anamalai Tiger Reserve C. Silent Valley National Park D. Wayanad Reserve Forests

Your Answer : Correct Answer : B

Answer Justification :

Learning: Earlier known as Indira Gandhi Wildlife Sanctuary and National Park (IGWLS&NP) and previously as Anamalai Wildlife Sanctuary, it is a protected area located in the Anaimalai Hills of Coimbatore District of Tamil Nadu state.

The park is named after former Prime Minister of India Indira Gandhi. The main tourist facilities are located in the northeast corner of the park at "Topslip", so named because of the local 19th century practice of sliding timber logs down the hills from here. prelims.insightsonindia.com 62 © Insights Active Learning | All rights reserved - 133617. You may not reproduce, distribute or exploit the contents in any form without written permission by copyright owner. Copyright infringers may face civil and criminal liability Online Prelims Thematic TEST - 10 ( InsightsIAS Mock Test Series for UPSC Preliminary Exam 2020 )

This tiger reserve, together with the several other contiguous protected forest and grassland habitats, is the core of the Parambikulum-Indira Gandhi tiger habitat landscape complex.

Q Source: Based on UPSC CSP papers

84 Which one of the following statements about the Treaty of Allahabad is correct?

A. It was concerned with the Battle of Buxar. B. It was imposed by force on the incumbent Mughal Emperor and deprived the Emperor of his property and land in lieu of protection. C. The rights given by this treaty allowed the Company to collect revenue directly from the people certain provinces. D. Both (a) and (c)

Your Answer : Correct Answer : D

Answer Justification :

Justification: It was signed in 1765, between the Mughal Emperor Shah Alam II, and Robert, Lord Clive as a result of the Battle of Buxar of 1764.

The Treaty marks the political and constitutional involvement and the beginning of British rule in India. Based on the terms of the agreement, Alam granted the East India Company Diwani rights, or the right to collect taxes on behalf of the Emperor from the eastern province of Bengal-Bihar- Orissa. Thus East India Company got appointed as the imperial tax collector for the Eastern province (Bengal-Bihar-Orissa). These rights allowed the Company to collect revenue directly from the people of Bengal, Bihar and Orissa. In return, the Company paid an annual tribute of twenty-six lakhs of rupees (equal IMAGERUNNERSto 260,000 pounds sterling) 8860450330 while securing for Shah Alam II the districts of Kora and Allahabad.

The tribute money paid to the emperor was for the maintenance of the Emperor's court in Allahabad.

Q Source: AR: UPSC CAPF 2017

85 Horse latitudes are characterized by

A. Calm winds B. Cloudy skies throughout the year C. Precipitation equivalent to the equatorial regions D. All of the above

Your Answer : prelims.insightsonindia.com 63 © Insights Active Learning | All rights reserved - 133617. You may not reproduce, distribute or exploit the contents in any form without written permission by copyright owner. Copyright infringers may face civil and criminal liability Online Prelims Thematic TEST - 10 ( InsightsIAS Mock Test Series for UPSC Preliminary Exam 2020 )

Correct Answer : A

Answer Justification :

Justification: The horse latitudes are located at about 30 degrees north and south of the equator.

It is common in this region of the subtropics for winds to diverge and either flow toward the poles (known as the prevailing westerlies) or toward the equator (known as the trade winds). These diverging winds are the result of an area of high pressure, which is characterized by calm winds, sunny skies, and little or no precipitation. According to legend, the term comes from ships sailing to the New World that would often become stalled for days or even weeks when they encountered areas of high pressure and calm winds. Many of these ships carried horses to the Americas as part of their cargo. Unable to sail and resupply due to lack of wind, crews often ran out of drinking water. To conserve scarce water, sailors on these ships would sometimes throw the horses they were transporting overboard. Thus, the phrase 'horse latitudes' was born.

Q Source: Based on UPSC CSP papers

86 Consider the following about Global Climate Observing System (GCOS). 1. Access to GCOS observations are only limited to member nations. 2. GCOS IMAGERUNNERS reports on the adequacy 8860450330 of the current climate observing system to the UNFCCC. Which of the above is/are correct? A. 1 only B. 2 only C. Both 1 and 2 D. None

Your Answer : Correct Answer : B

Answer Justification :

Justification: As an outcome of the Second World Climate Conference, the Global Climate Observing System (GCOS) was established in 1992 to ensure that the observations and information needed to address climate-related issues are obtained and made available to all potential users.

prelims.insightsonindia.com 64 © Insights Active Learning | All rights reserved - 133617. You may not reproduce, distribute or exploit the contents in any form without written permission by copyright owner. Copyright infringers may face civil and criminal liability Online Prelims Thematic TEST - 10 ( InsightsIAS Mock Test Series for UPSC Preliminary Exam 2020 )

The GCOS is co-sponsored by the World Meteorological Organization (WMO), the Intergovernmental Oceanographic Commission (IOC) of UNESCO, the United Nations Environment Programme (UNEP), and the International Council for Science (ICSU). In order to assess and monitor the adequacy of in-situ observation networks as well as satellite-based observing systems, GCOS regularly reports on the adequacy of the current climate observing system to the United Nations Framework Convention on Climate Change (UNFCCC), and thereby identifies the needs of the current climate observing system. The vision of Global Climate Observing System (GCOS) is for all users to have access to the climate observations, data records and information they need to address pressing climate-related concerns. GCOS users include individuals, national and international organizations, institutions and agencies.

Image: Measurements of air temperature, a GCOS ECV, provided by the Copernicus Climate Change Service (C3S), implemented by the European Centre for Medium-Range Weather Forecasts (ECMWF).

See IMAGERUNNERS https://public.wmo.int/en/programmes/global-climate-observing-system 8860450330

http://unfccc.int/science/knowledge_resources/external_resources/items/7539.php

Q Source: Based on CSP papers

87 Which of these water bodies is the richest in coral species?

A. Mediterranean Sea B. Atlantic Ocean C. Indian Ocean D. Pacific Ocean

Your Answer : Correct Answer : D prelims.insightsonindia.com 65 © Insights Active Learning | All rights reserved - 133617. You may not reproduce, distribute or exploit the contents in any form without written permission by copyright owner. Copyright infringers may face civil and criminal liability Online Prelims Thematic TEST - 10 ( InsightsIAS Mock Test Series for UPSC Preliminary Exam 2020 )

Answer Justification :

Learning: Reef-building corals are restricted in their geographic distribution by factors such as the temperature and the salinity (salt content) of the water. The water must also be clear to permit high light penetration.

Because of these environmental restrictions, reefs generally are confined to tropical and semitropical waters. The diversity of reef corals (the number of species), decreases in higher latitudes up to about 30° north and south, beyond which reef corals are usually not found.

Generally, there are about twice as many coral species in Pacific Ocean reefs as in Atlantic Ocean reefs.

See https://oceanservice.noaa.gov/facts/most_coral.html

Q Source: Based on UPSC CSP papers

88 Action for Climate Empowerment (ACE) is a term adopted in which of following important climate change mitigation treaties/protocols/conventions?

A. Montreal Protocol B. Convention on Biological Diversity C. UNFCCC D. Paris convention 2015

Your Answer : Correct Answer : C

Answer Justification :

Learning: It is a part of UNFCCC and refers to Article 6 of the Convention’s original text (1992), focusing on six priority areas: education, training, public awareness, public participation, public access IMAGERUNNERS to information, and international 8860450330 cooperation on these issues.

The implementation of all six areas has been identified as the pivotal factor for everyone to understand and participate in solving the complex challenges presented by climate change.

ACE calls on governments to develop and implement educational and public awareness programmes, train scientific, technical and managerial personnel, foster access to information, and promote public participation in addressing climate change and its effects.

It also urges countries to cooperate in this process, by exchanging good practices and lessons learned, and strengthening national institutions.

Q Source: Based on UPSC CSP papers

89 Consider the following about this ruler in Medieval India. 1. His rule is considered significant by historians for the introduction of token currency. prelims.insightsonindia.com 66 © Insights Active Learning | All rights reserved - 133617. You may not reproduce, distribute or exploit the contents in any form without written permission by copyright owner. Copyright infringers may face civil and criminal liability Online Prelims Thematic TEST - 10 ( InsightsIAS Mock Test Series for UPSC Preliminary Exam 2020 )

2. He took keen interest to circulate gold and silver coins and introduced the gold coin as Dinar in India.

The above refer to? A. Iltutmish B. Mir Kasim C. Tughlaq D. Aurangjeb

Your Answer : Correct Answer : C

Answer Justification :

Learning: He understood the importance of currency as a medium of commercial exchange and that is why he took keen interest to circulate gold and silver coins.

The gold coin was introduced as Dinar. Tughlaq’s silver coin was named Adl. However, it was difficult to maintain the supply of gold and silver coins on a large scale.

So, Tughlaq replaced those coins and started the circulation of copper and brass coins as the token currency which had the same value of gold or silver coins in 1330-32 CE.

He was well aware that the state had to act as a responsible guarantor for the token money by ensuring high degree of security which will prevent others from making fake currencies.

Q Source: Based on UPSC CSP papers

https://www.coin-competition.eu/history/the-coins-of-muhammad-bin-tughlaq/

90 BSE IndoNext was primarily formed to promote

A.IMAGERUNNERSBusiness process outsourcing 8860450330 (BPO) units B. Unorganized sector C. Bluechip companies D. Small and medium size companies

Your Answer : Correct Answer : D

Answer Justification :

Background and Learning: Investors and the companies, which are listed only on Regional Stock Exchanges (RSEs) are impacted severely, as many RSEs are recording either nil or negligible trading volumes.

Investors in such small and medium companies are therefore unable to find an exit route and realize fair market value of their investments. prelims.insightsonindia.com 67 © Insights Active Learning | All rights reserved - 133617. You may not reproduce, distribute or exploit the contents in any form without written permission by copyright owner. Copyright infringers may face civil and criminal liability Online Prelims Thematic TEST - 10 ( InsightsIAS Mock Test Series for UPSC Preliminary Exam 2020 )

The companies listed only at RSEs find it extremely difficult, if not impossible, to raise fresh resources from the capital markets in the absence of price discovery of their securities in the secondary market. New small and medium enterprises (SMEs) have also been unable to tap the markets to raise equity and debt, to fund their plans. It is to solve this problem that IndoNext was launched.

Q Source: Based on UPSC CSP papers

http://www.bseindia.com/Static/about/indonext.aspx

91 With reference to the United Nations Framework Convention on Climate Change (UNFCCC), consider the following statements. 1. It is the parent treaty of the 1997 Kyoto Protocol. 2. The framework sets binding limits on greenhouse gas emissions for individual countries. 3. It contains no legal/binding enforcement mechanisms for the provisions of the convention.

Select the correct answer using the codes below. A. 1 only B. 2 and 3 only C. 1 and 3 only D. 1, 2 and 3

Your Answer : Correct Answer : C

Answer Justification :

Justification: With 197 Parties, the United Nations Framework Convention on Climate Change (UNFCCC) has near universal membership and is the parent treaty of the 1997 Kyoto Protocol. The Kyoto Protocol has been ratified by 192 of the UNFCCC Parties.

IMAGERUNNERSThe ultimate objective of 8860450330 both treaties is to stabilize greenhouse gas concentrations in the atmosphere at a level that will prevent dangerous human interference with the climate system. It is an international environmental treaty adopted in 1992 at the Earth Summit in Rio de Janeiro.

Statement 2: The framework sets non binding limits on greenhouse gas emissions for individual countries and contains no enforcement mechanisms. Instead, the framework outlines how specific international treaties (called "protocols" or "Agreements") may be negotiated to specify further action towards the objective of the UNFCCC.

One of the first tasks set by the UNFCCC was for signatory nations to establish national greenhouse gas inventories of greenhouse gas (GHG) emissions and removals, which were used to create the 1990 benchmark levels for accession of Annex I countries to the Kyoto Protocol and for the commitment of those countries to GHG reductions.

prelims.insightsonindia.com 68 © Insights Active Learning | All rights reserved - 133617. You may not reproduce, distribute or exploit the contents in any form without written permission by copyright owner. Copyright infringers may face civil and criminal liability Online Prelims Thematic TEST - 10 ( InsightsIAS Mock Test Series for UPSC Preliminary Exam 2020 )

Updated inventories must be submitted annually by Annex I countries.

Q Source: Based on UPSC CSP papers

92 Amongst the following, this source contains the least nutritional calcium per 100 g?

A. Rice B. Ragi C. Skimmed milk D. Egg

Your Answer : Correct Answer : A

Answer Justification :

Justification: Ragi is one of the best non-dairy sources of calcium when compared to any other grains. According to the National Institute of Nutrition in India, 100 grams of Ragi contains 344 mg calcium.

In comparison, Rice contans only 10 mg of calcium per 100g.

Skimmed milk and eggs are good sources of calcium.

Q Source: Based on UPSC CSP papers

93 With reference to Asian Infrastructure and Investment Bank (AIIB), consider the following statements. 1. It offers sovereign and non-sovereign financing for sustainable projects in energy, telecommunications and rural infrastructure. 2. It may participate in the underwriting of securities issued by any entity or for purposes consistent IMAGERUNNERS with its purpose. 8860450330

Which of the above is/are correct? A. 1 only B. 2 only C. Both 1 and 2 D. None

Your Answer : Correct Answer : C

Answer Justification :

Justification: According to the Articles of Agreement (AOA) of AIIB, the Bank will “provide or facilitate financing to any member, or any agency, instrumentality or political subdivision thereof, or any entity or enterprise operating in the territory of a member, as well as to international or prelims.insightsonindia.com 69 © Insights Active Learning | All rights reserved - 133617. You may not reproduce, distribute or exploit the contents in any form without written permission by copyright owner. Copyright infringers may face civil and criminal liability Online Prelims Thematic TEST - 10 ( InsightsIAS Mock Test Series for UPSC Preliminary Exam 2020 )

regional agencies or entities concerned with economic development of the Asia region.”

Furthermore, the AOA permits the Bank to provide financing in a variety of ways, including, inter alia, making loans, investing in the equity capital of an enterprise, and guaranteeing, whether as primary or secondary obligor, in whole or in part, loans for economic development.

In addition, the Bank may underwrite, or participate in the underwriting of, securities issued by any entity or enterprise for purposes consistent with its purpose.

Other areas of Bank finance include agriculture development, water supply and sanitation, environmental protection, and urban development and logistics.

Q Source: AR: UPSC past year papers

94 Consider the following statements. 1. Dead sea is the world’s highest water body in terms of mean altitude. 2. Lake Tanganyika is World’s lowest situated lake. 3. Caspian Sea, which is a major water body in Eurasia, is a tectonic lake.

Select the correct answer using the codes below. A. 1 and 2 only B. 2 and 3 only C. 3 only D. None of the above

Your Answer : Correct Answer : C

Answer Justification :

Justification: Statement 1 and 2: Due to faulting, a rift valley is formed by the sinking of land between two parallel faults, deep, narrow and elongated in character. Water collects in these troughs IMAGERUNNERS and their floors are often 8860450330 below sea level.

It includes such lakes as Lakes Tanganyika (4,700 feet deep, the world’s deepest lake), Malawi, Rudolf, Edward. Albert, as well as the Dead Sea 1,286 feet below mean sea level, the world’s lowest lake. The best known example is the East African Rift Valley which runs through Zambia, Malawi, Tanzania, Kenya and Ethiopia, and extends along the Red Sea to Israel and Jordan over a total distance of 3,000 miles.

S3: Due to the warping, sagging, bending and fracturing of the earth’s crust, tectonic depressions occur. Such depressions give rise to lakes of immense sizes and depths.

They include Lake Titicaca, occupying a huge depression in the intermont plateau of the Andes, 12,500 feet above sea level the highest lake in the world; and the Caspian Sea, 143,550 square miles, the largest lake, almost 5 times larger than its nearest rival, Lake Superior.

Q Source: Based on CSP papers prelims.insightsonindia.com 70 © Insights Active Learning | All rights reserved - 133617. You may not reproduce, distribute or exploit the contents in any form without written permission by copyright owner. Copyright infringers may face civil and criminal liability Online Prelims Thematic TEST - 10 ( InsightsIAS Mock Test Series for UPSC Preliminary Exam 2020 )

95 Arrange the following Continents in the decreasing order of their Geographical area :

A. Asia - Africa - South America - Europe B. South America - Asia – Africa - Europe C. Africa - Asia – South America - Europe D. Asia – Africa - Europe - South America

Your Answer : Correct Answer : A

Answer Justification :

Justification: Please see illustration below.

Q Source: Based on CSP papers

96 The Partnership IMAGERUNNERS for Peace (PfP) 8860450330is an initiative primarily concerned with which of these organizations?

A. NATO B. European Union C. United Nations Security Council D. Warsaw Pact

Your Answer : Correct Answer : A

Answer Justification :

Learning: The Partnership for Peace (PfP) is a programme of practical bilateral cooperation between individual Euro-Atlantic partner countries and NATO. It allows partners to build up an individual relationship with NATO, choosing their own priorities for cooperation. prelims.insightsonindia.com 71 © Insights Active Learning | All rights reserved - 133617. You may not reproduce, distribute or exploit the contents in any form without written permission by copyright owner. Copyright infringers may face civil and criminal liability Online Prelims Thematic TEST - 10 ( InsightsIAS Mock Test Series for UPSC Preliminary Exam 2020 )

It was first discussed by the Bulgarian Society Novae, after being proposed as an American initiative at the meeting of NATO defence ministers in Germany, in 1993, and formally launched in 1994 NATO summit in Brussels.

NATO builds relationships with partners through military-to-military cooperation on training, exercises, disaster planning and response, science and environmental issues, professionalization, policy planning, and relations with civilian government.

Q Source: Based on UPSC CSP papers

97 Sound travels the fastest in

A. A gas with low density B. A liquid with high density C. A solid with extremely low density D. A solid with extremely high density

Your Answer : Correct Answer : D

Answer Justification :

Justification: The speed of sound in an ideal gas depends only on its temperature and composition.

The speed has a weak dependence on frequency and pressure in ordinary air, deviating slightly from ideal behaviour. However, the speed of sound varies from substance to substance: sound travels most slowly in gases; it travels faster in liquids; and faster still in solids. For example, (as noted above), sound travels at 343 m/s in air; it travels at 1,484 m/s in water (4.3 times as fast as in air); and at 5,120 m/s in iron (about 15 times as fast as in air). In an exceptionally stiff material such as diamond, sound travels at 12,000 metres per second IMAGERUNNERS(26,843 mph); (about 35 8860450330 times as fast as in air) which is around the maximum speed that sound will travel under normal conditions.

Q Source: Based on UPSC CSP papers

98 The Heavy Mineral Sand deposits in Kerala contains which of these minerals? 1. Ilmenite 2. Zircon 3. Monazite 4. Rutile

Select the correct answer using the codes below. A. 1 and 2 only B. 2, 3 and 4 only C. 1, 3 and 4 only D. 1, 2, 3 and 4 prelims.insightsonindia.com 72 © Insights Active Learning | All rights reserved - 133617. You may not reproduce, distribute or exploit the contents in any form without written permission by copyright owner. Copyright infringers may face civil and criminal liability Online Prelims Thematic TEST - 10 ( InsightsIAS Mock Test Series for UPSC Preliminary Exam 2020 )

Your Answer : Correct Answer : D

Answer Justification :

Justification: Kerala State is endowed with a number of occurrences/deposits of minerals such as Heavy Mineral Sands ( Ilmenite, Rutile, Zircon, Monazite, Sillimanite) ,Gold, Iron ore, Bauxite, Graphite, China Clay, Fire Clay, Tile and Brick Clay, Silica Sand, Lignite, Limestone, Limeshell, Dimension Stone (Granite), Gemstones, Magnesite, Steatite etc.

However, mining activities on large scale are confined mainly to a few minerals - Heavy Mineral Sands, China Clay and to a lesser extent Limestone/Limeshell, Silica Sand and Granite.

In fact, Heavy mineral sand and China Clay contribute more than 90% of the total value of mineral production in the State.

Q Source: Based on UPSC CSP papers

99 The recommendations of this conference/meeting were published in a White Paper, 1933, which became the basis of the Government of India Act 1935:

A. Macaulay minutes B. Third Round Table Conference under McDonald C. Chelmsford Committee proceedings D. Wavell Conference IMAGERUNNERS 8860450330 Your Answer : Correct Answer : B

Answer Justification :

Learning: In response to the inadequacy of the Simon Report, the Labour Government, which had come to power under Ramsay MacDonald in 1929, decided to hold a series of Round Table Conferences in London.

The third Round Table Conference (17 November 1932 - 24 December 1932) was not attended by the Indian National Congress and Gandhi. Many other Indian leaders were also absent. Like the two first conferences, little was achieved.

The recommendations were published in a White Paper in March 1933 and debated in Parliament afterwards. A Joint Select Committee was formed to analyse the recommendations and formulate a new Act for India. prelims.insightsonindia.com 73 © Insights Active Learning | All rights reserved - 133617. You may not reproduce, distribute or exploit the contents in any form without written permission by copyright owner. Copyright infringers may face civil and criminal liability Online Prelims Thematic TEST - 10 ( InsightsIAS Mock Test Series for UPSC Preliminary Exam 2020 )

The Committee produced a draft Bill in February 1935 which was enforced as the Government of India Act of 1935 in July 1935.

Q Source: Based on past CSP papers

100 Consider the following about NABSKILL. 1. It is a digital interface to promote Skill INDIA. 2. It is an initiative of the National Skill Development Corporation of India.

Which of the above is/are correct? A. 1 only B. 2 only C. Both 1 and 2 D. None

Your Answer : Correct Answer : A

Answer Justification :

Learning: The portal capture the details of all stakeholders (Training Seeker, Training Provider, Placement Agencies and NABARD) involved in skill development initiatives of NABARD.

The eligible training provider can check for their eligibility criteria under “Guidelines” in home page and upon successful registration the training provider can apply online for seeking grant support from NABARD.

The portal captures the details of training related data viz., trainees profiles, training programme details, post training settlement rate etc.

The registered placement agencies can view the details of successful trainees (Name, Type of training obtained, Details of Training Institute) who have undergone skill training supported by NABARD. IMAGERUNNERS The placement agencies 8860450330 can approach the training providers for further information on placement, etc.

prelims.insightsonindia.com 74 © Insights Active Learning | All rights reserved - 133617. You may not reproduce, distribute or exploit the contents in any form without written permission by copyright owner. Copyright infringers may face civil and criminal liability Online Prelims Thematic TEST - 10 ( InsightsIAS Mock Test Series for UPSC Preliminary Exam 2020 )

See http://www.nabskillnabard.org/aboutus.php?language=LG-1&status=Active

Q Source: AR: UPSC past year papers

IMAGERUNNERS 8860450330

prelims.insightsonindia.com 75 © Insights Active Learning | All rights reserved - 133617. You may not reproduce, distribute or exploit the contents in any form without written permission by copyright owner. Copyright infringers may face civil and criminal liability